Você está na página 1de 129

DAMS CBT 2016 SET - 1

Q-1 Renal fascia is also known as?


a. Scarpas fascia
b. Gerotas fascia
c. Denonvilliers fascia
d. Bucks fascia
Ans-1: (b) Gerotas fascia
Ref.:Read the text below
Sol :
Bucks fascia
Colless fascia
Denonvilliers fascia
Gerotas fascia
Scarpas fascia
Zuckerkandls fascia

The penile fascial sheath


Continuation of scarpas layer of abdominal fascia.
Fascia separating the prostate from the rectum
The renal fascia; encapsulates the perinephric fat.
The fibrous layer of the superficial fascia of the lower
abdomen
The retrorenal fascia.

Q-2 Inferior mesenteric artery does not supply?


a. Rectum
b. Descending colon
c. Right flexure of transverse colon
d. Upper part of the anal canal
Ans-2: (c) Right flexure of transverse colon
Ref.:Read the text below
Sol :
Right flexure of transverse colon is supplied by superior mesenteric artery not inferior
mesenteric artery
Part
Part in adult
Gives rise to
Arterial Supply
Foregut
Esophagus to first
Esophagus, Stomach,
Celiac trunk
nd
2 sections of the
Duodenum first and 2 parts),
duodenum
Liver, Gallbladder, Pancreas,
spleen, Superior portion of
pancreas
Midgut
Lower duodenum, to
Lower duodenum, jejunum,
Branches of the
the first two-thirds of ileum, cecum, appendix,
superior
the transverse colon
ascending colon, and first two
mesenteric artery
thirds of the transverse colon
Hindgut
Last third of the
Last third of the transverse
Branches of the
transverse colon, to
colon, descending colon,
inferior mesenteric
the upper part of the rectum, and upper part of the
artery
anal canal
anal canal

www.damsdelhi.com

Email: info@damsdelhi.com

DAMS CBT 2016 SET - 1

Q-3 Regarding meningeal blood supply and innervations,incorrect statement is:


a. The main blood supply to the meninges is from the middle meningeal artery.
b. The accessory meningeal artery may arise from the maxillary or middle meningeal
artery.
c. The middle meningeal artery is subdural in location.
d. Innervation of the dura is primarily from the trigeminal nerve.
Ans 3: (c) The middle meningeal artery is subdural in location.
Reference Read the text below
Sol:
The main blood supply to the meninges is from the middle meningeal artery however,
there are contributions from the cavernous, carotid,ophthalmic and vertebral arteries.
The accessory meningeal artery enters the skull through the foramen ovale and
supplies the meninges.
The middle meningeal artery is extradural and along with the meningeal veins grooves
the inner table of the skull.
Innervation of the dura is primarily from the trigeminal nerve and also from the lower
cranial nerves and the first three cervical segments. This may be the reason for cervical pain
in cranial subarachnoid haemorrhage.

Q-4 Regarding the normal development of the brain:


a. The neural tube expands to form the three primary vesicles during the fourth week of
intrauterine development.
b. Hypodense white matter in a pre-term infants CT brain usually signifies ischaemia.
c. Vascular supply of the embryonic brain is similar to that of the adult.
d. MRI is insensitive to assess the progress of myelination.
Ans 4: (a) The neural tube expands to form the three primary vesicles during the fourth
week of intrauterine development.
Reference Read the text below
Sol:
Due to the relatively high water content, white matter of the normal preterm infant
appears hypodense and should not be mistaken for ischaemia.
The embryonic brain is exclusively supplied by the internal carotid artery, which may
persist in the adult when one of the two posterior cerebral arteries is supplied only through
the ipsilateral posterior communicating artery.
MRI is used to assess the progress of myelination. T1-weighted inversion recovery
images are particularly sensitive to myelination in the first 6 months. Thereafter T2 weighted images are used.

Q-5 Lateral cutaneous nerve of thigh is a branch of?


a. Lumbar plexus
b. Femoral nerve
c. Obturator nerve
d. Sciatic nerve

www.damsdelhi.com

Email: info@damsdelhi.com

DAMS CBT 2016 SET - 1

Ans-5: (a) Lumbar plexus


Ref.:Read the text below
Sol :
Lateral cutaneous nerve of thigh is a purely sensory branch of lumbar plexus L2 L3 as it
passes through the opening between inguinal ligament and its attachment 1-2 cm medial to
the anterior superior iliac spine.
Entrapment of lateral cutaneous nerve of thigh causes Meralgia paresthetica.
Numbness is the earliest and most common symptom
Pain, paresthesia, Hyposthesia over the anteriolateral aspect of thigh.
Particularly common in obese and in abdominal distension.

Q-6 Bochdalek hernia is?


a. Anterolateral hiatus hernia
b. Posterolateral hiatus hernia
c. Central hiatus hernia
d. Anterior hiatus hernia
Ans-6: (b) Posterolateral hiatus hernia
Ref.:Read the text below
Sol :
The classic appearance of a hiatus hernia, with a fluid level superimposed on the
cardiac shadow on the PA film, is well known.
A bochdalek hernia arise posterolaterally through the pleuroperitoneal canal and is
usually congenital, presenting at birth as respiratory distress.
Ninety percent are left sided.
The hernia may contain omentum, fat, spleen, kidney and bowel, in which case a gas
shadow is seen within the mass.
The ipsilateral lung is invariably hypoplastic with deviation of the mediastinum away
from the side of the hernia.

Q-7 Which of the following best describes the landmark known as lambda?
a. Point on calvaria at junction of sagittal and lambdoid sutures
b. Point on calvaria at junction of sagittal and coronal sutures
c. Junction of the greater wing of the sphenoid, squamous temporal, frontal, and parietal
bones
d. Star-shaped landmark at junction of parietomastoid,occipitomastoid, and lambdoid
sutures
Ans 7: (a) Point on calvaria at junction of sagittal and lambdoid sutures
Ref Read the text below
Sol:

www.damsdelhi.com

Email: info@damsdelhi.com

DAMS CBT 2016 SET - 1

Lambda is the point on the calvaria at the junction of the sagittal and lambdoid
sutures

Q-8 Which of the following transmits a nerve branch to the meninges?


a. Foramen spinosum
b. Mastoid foramen
c. Jugular foramen
d. Foramen magnum
Ans 8: (a) Foramen spinosum
Ref Read the text below
Sol:
The foramen spinosum transmits the meningeal branch of the mandibular division of
the trigeminal nerve

Q-9 Theforamen magnum transmits all of the following except


a. Medulla and meninges
b. Vertebral arteries
c. Spinal roots of the accessory nerve
d. Internal carotid artery
Ans 9: (d) Internal carotid artery
Ref Read the text below
Sol:
The foramen magnum transmits the medulla and meninges, vertebral arteries, spinal
roots of the accessory nerve, dural veins, and the anterior and posterior spinal arteries

www.damsdelhi.com

Email: info@damsdelhi.com

DAMS CBT 2016 SET - 1

Q-10 All are supplied by mandibular nerve except:


a. Anterior belly of digastrics
b. Omohyoid
c. Tensor tympani
d. Tensor veli palatine
Ans-10: (b) Omohyoid
Ref.:Read the text below
Sol :
The superior belly of omohyoid is innervated by branches from the superior ramus of
the ansa cervicalis (C1).
The inferior belly is innervated from the ansa cervicalis itself (C1, 2 and 3)
The anterior belly of digastric is supplied by the mylohyoid branch of the inferior
alveolar nerve (post division of mandibular), and the posterior belly is supplied by the
facial nerve.

Q-11 Pleural reflection on left mid axillary line is upto which intercostals space?
a. 5
b. 6
c. 8
d. 10
Ans-11: (d) 10
Ref.:Read the text below
Sol :
The inferior border is thin and sharp where it separates the base from the costal
surface and extends into the costo diaphragmatic recess.
It is more rounded medially where it divides the base from the mediastinal surface.
The lower border of the pleura on both sides follows a curved line, which crosses the 8 th
rib in midclavicular line, and the 10th rib in midaxillary line, and reaches the 12 th rib
adjacent to the vertebral column, that is at the lateral border of erector spinae muscle
Note :
Lower margins of lungs crosses the 6th , 8th and 10 ribs at the midclavicular lines, and
axillary lines and sides of the vertebral column respectively.
The distance between the lower border of lungs and plura corresponds the
costodiaphrangmatic recess.

Q-12 A patient is unable to prevent anterior displacement of the femur on the tibia when
the knee is flexed. Which of the following ligaments is most likely damaged?
a. Anterior cruciate
b. Fibular collateral
c. Patellar
d. Posterior cruciate
Ans 12: (d) Posterior cruciate
www.damsdelhi.com

Email: info@damsdelhi.com

DAMS CBT 2016 SET - 1

Reference Read the text below


Sol:
The posterior cruciate ligament is important because it prevents forward displacement
of the femur on the tibia when the knee is flexed.
The anterior cruciate ligament prevents backward displacement of the femur on the
tibia.

Q-13 To avoid damaging the sciatic nerve during an injection, the needle should be
inserted into which of the following areas?
a. Over the sacrospinous ligament
b. Midway between the ischial tuberosity and the lesser trochanter
c. Midpoint of the gemelli muscles
d. Upper lateral quadrant of the gluteal region
Ans 13: (d) Upper lateral quadrant of the gluteal region
Reference Read the text below
Sol:
To avoid damaging the sciatic nerve during an intramuscular injection, the clinician
should insert the needle in the upper lateral quadrant of the gluteal region.
The inserted needle in the lower medial quadrant may damage the pudendal and sciatic
nerves.
The inserted needle midway between the ischial tuberosity and the lesser trochanter
may damage the sciatic and posterior femoral cutaneous nerves on the quadratus femoris.
The inserted needle over the sacrospinous ligament may damage the pudendal nerve
and vessels.

Q-14 A 20-year-old patient cannot flex and medially rotate the thigh while running and
climbing. Which of the following muscles is most likely damaged?
a. Semimembranosus
b. Rectus femoris
c. Vastus intermedius
d. Tensor fasciae latae
Ans 14: (d) Tensor fasciae latae
Reference Read the text below
Sol:
The tensor fasciae latae can flex and medially rotate the thigh, so this is the muscle
most likely damaged.
The hamstring muscles (semitendinosus, semimembranosus, and biceps femoris) can
extend the thigh and flex the leg.
The sartorius can flex the thigh and leg. The rectus femoris can flex the thigh and
extend the leg.
The vastus intermedius can extend the leg.

www.damsdelhi.com

Email: info@damsdelhi.com

DAMS CBT 2016 SET - 1

Q-15 A 22-year-old patient is unable to unlock the knee joint to permit flexion of the leg.
Which of the following muscles is most likely damaged?
a. Rectus femoris
b. Semimembranosus
c. Popliteus
d. Gastrocnemius
Ans 15: (c) Popliteus
Reference Read the text below
Sol:
The popliteus muscle rotates the femur laterally (unlocks the knee) or rotates the tibia
medially, depending on which bone is fixed.
This action results in unlocking of the knee joint to initiate flexion of the leg at the
joint. The rectus femoris flexes the thigh and extends the knee.
The gastrocnemius flexes the knee and plantar flexes the foot. The semimembranosus
extends the thigh and flexes and rotates the leg medially.
The biceps femoris extends the thigh and flexes and rotates the leg laterally.
Q-16 Planning and programming of movements is done by which part of brain?
a. Vestibulocerebellum
b. Motor cortex
c. Spinocerebellum
d. Basal ganglia
Ans-16: (d) Basal ganglia
Ref.: Read the text below
Sol :
Planning and programming of movements are done by both cerebrocerebellum and
basal ganglia.
Studies have made it clear that neurons in the basal ganglia, like those in the lateral
portions of the cerebellar hemispheres, discharge before movements begin.
These observations, plus careful analysis of the effects of diseases of the basal ganglion
in humans and the effects of drugs that destroy dopaminergic neurons in animals, have led
to the concept that the basal ganglia are involved in the planning and programming of
movement or, more broadly, in the processes by which an abstract thought is converted
into voluntary action.

Q-17 Duration of second heart sound is?


a. 0.12 sec
b. 0.1 sec
c. 0.15 Sec
d. 02. Sec
Ans-17: (a) 0.12 sec
Ref.:Read the text below
Sol :
Heart
Duration
Pitch or
Character

www.damsdelhi.com

Cause

Timing

Email: info@damsdelhi.com

DAMS CBT 2016 SET - 1

sound
1st

0.15 sec

frequency
25 to 45 Hz

High pitched

2nd

0.12 sec

50 Hz

Higher than S1

3rd

0.1 sec

--

Soft

4th

--

<20 Hz

Soft

Sudden
closure of
mitral and
tricuspid
valves
Closure of
aortic or
pulmonary
valves
Rapid
ventricular
filling due to
blood
inrushing from
atria
Filling of
ventricles due
to aortic
contraction

Start of
ventricular
systole

Just after end of


ventricular
systole
Beginning of
middle third of
diastole

Just Before 1st


sound

Q-18 The term Mileu interior was given by?


a. Knut schmidt-nielsen
b. George Bartholomew
c. Claude Bernard
d. Walter Cannon
Ans-18: (c) Claude Bernard
Ref.:Read the text below
Sol :
Milieu intrieur or interior milieu, from the French, milieu intrieur (the environment
within), is a phrase coined by Claude Bernard to refer to the extra-cellular fluid
environment, and its physiological capacity to ensure protective stability for the tissues and
organs of multicellularliving organisms.

Q-19 Avoiding response is due to which of the following lesion in the brain.
a. Ipsilateral frontal lobe lesions
b. Contralateral frontal lobe lesions
c. Ipsilateral parietal lobe lesions
d. Contralateral parietal lobe lesions
Ans-19: (d) Contralateral parietal lobe lesions
Ref.:Read the text below
Sol :

www.damsdelhi.com

Email: info@damsdelhi.com

DAMS CBT 2016 SET - 1

In the demented patient and in other organic confusional states, certain reflexes,
released from the control of higher center, may be elicited.
Some may also be found in patients with large focal lesions ; for example, the grasp
reflex is characteristically released in patients with contra lateral frontal lobe disease.
In infancy the presence or absence of these reflexes is used as part of the development
assessment.
The most important of these higher level reflexes are the grasping and avoiding
responses the palmomental reflex, the glabellar tap reflex released oral responses such as
the snout response and tactilely and visually evoked sucking reflexes.
The avoiding response consists of a tendency for the patients hand to move away from
palmar or dorsal contact.
It is usually evoked by stimuli on the ulnar side of the hand.
It is released in patients with contralateral parietal lobe disease, or lesions in is
connections.
Q-20 SODIUM Na+ Reabsorption in association with secretion of K+in a normal male
when GFR = 125 mL/min ,Plasma HCO3- = 27 meq/L; Plasma Na+ =145 meq/L is :
a. 5 ueq
b. 50 ueq
c. 500 ueq
d. 1000 ueq
Ans-20: (b) 50 ueq
Ref.:Read the text below
Sol :
Quantitative aspects of Na reabsorption in a
Normal man
GFR = 125 mL/min; Plasma HCO = 27 meq/L; Plasma Na =
145 meq/L
Na+ filtered per minute
18,125 ueq.
Reabsorbed with CI14,585 ueq.
Reabsorbed while reabsorbing 3375 ueq of HCO3
3,375 ueq.
Reabsorbed in association with formation of filterable
50 ueq.
acidity and ammonia
Reabsorbed in association with secretion of K+
50 ueq.
Total Na+ reabsorbed per minute
18,060 ueq.

Q-21 In gametogenesis, meiosis differs significantly from mitosis. All of the following are
characteristics of meiosis except :
a. Crossing over occurs between sister chromatids during prophase I
b. Daughter cells enter a second M phase of the cell cycle after division I without passing
through the GI phase and S phase
c. Independent assortment of maternal and paternal genetic material occurs in division I
d. The result of division I is a chromosomal reduction to the haploid number.
Ans-21: (d) The result of division I is a chromosomal reduction to the haploid number.
www.damsdelhi.com

Email: info@damsdelhi.com

DAMS CBT 2016 SET - 1

Ref.:Read the text below


Sol :
Gametogenesis uses meiosis to admix the parental genetic material and to produce a
haploid gamete.
There is pairing of homologous chromosomes and chromosomal duplication during
prophase I.
The paired homologues align in the metaphase and crossing over occurs between sister
chromatids.
Also at this stage the independent assortment of maternal and paternal genetic
material occurs. Because the chromosomal replication results in a tetraploid chromosomal
number, division I produces cells with the diploid number.
After division I the daughter cells enter a second M phase of the cell cycle without
passing through the GI phase and S phase. Thus, the result of division. It is a chromosomal
reduction to the haploid number.

Q-22 Which non proteinous substance has highest normal blood level?
a. Urea
b. Creatinine
c. Uric acid
d. Ammonia
Ans-22: (a) Urea
Ref.:Read the text below
Sol :
Substance
Normal Level
Urea
20-40 mg/dl
Creatinine
05.-0.9 mg/dL (female), 0.6-1.2 mg/dL (male)
Uric acid
2.5-5.6 mg/dL (female), 3.1 7.0 mg/dL (male)
Ammonia
19-60 ug/dL

Q-23 Plasma volume is best evaluated by using?


a. Evan blue
b. Inulin
c. Mannitol
d. Radiolabelled water
Ans-23: (a) EVAN'S BLUE
Ref.:Read the text below
Sol :
T-1824 or Evans Blue, often incorrectly rendered as Evan's Blue, is an azo dye which has a
very high affinity for serum albumin. Because of this, it can be useful in physiology in
estimating the proportion of body water contained in blood plasma

Q-24 Golgi tendons and muscle fibre ratio is?

www.damsdelhi.com

Email: info@damsdelhi.com

DAMS CBT 2016 SET - 1

a. 1:1
b. 1 : 3-25
c. 1 : 40
d. 1 : 50
Ans-24: (b) 1 : 3-25
Ref.:Read the text below
Sol :
This organ consists of a net-like collection of knobby nerve endings among the fascicles
of a tendon. There are 3-25 muscle fibers per tendon organ.
The fibers from the Golgi tendon organs make up the Ib group of myelinated, rapidly
conducting sensory nerve fibers.

Q-25 Blood pressure changes by ____ above or below heart level?


a. 0.55 mm of Hg/cm
b. 0.50 mm of Hg/cm
c. 0.60 mm of Hg/cm
d. 0.77 mm of Hg/cm
Ans-25: (d) 0.77 mm of Hg/m
Ref.:Read the text below
Sol :
The blood pressure increases or decreases by 0.77 mm of Hg/cm for every centimeter
below or above the level of heart.

Q-26 Hematocrit in veins is higher than arteries because?


a. Increased number of particles
b. Slow flow
c. Lymphatic absorption
d. Larger amount of blood
Ans-26: (a) Increased number of particles
Ref.:Read the text below
Sol :
Veins have higher hematocrit compared to arteries because due to addition of chloride
ions, bicarbonate ions and CO2 which makes blood osmolarity in veins to increase and
therefore red cells take in water and hematocrit in veins increase.
This is major reason, minor reason is formation of tissue fluid in capillaries & absorption
of some fluid as lymphatics.

Q-27 Action of pineal gland is?


a. Circadian rhythm
b. Secretion of TSH
c. Control of the impulses
d. Central control of autonomic nervous system.
www.damsdelhi.com

Email: info@damsdelhi.com

DAMS CBT 2016 SET - 1

Ans-27: (a) Circadian rhythm


Ref.:Read the text below
Sol :
Some pineal indoleamines, including melatonin and enzymes for their biosynthesis (e.g.
serotonin N-acetyltransferase) show circadian rhythms in concentration.
The level rises during darkness, and falls during the day, when secretion may be
inhibited by sympathetic activity. It is thought that the intrinsic rhythmicity of an
endogenous circadian oscillator in the suprachiasmatic nucleus of the hypothalamus
governs cyclical pineal behavior.

Q-28 All the following are characteristics of the plasma membrane except:
a. It serves as a selective filtration barrier through the facilitation of both active and
passive transport
b. It possesses a thickness of 1 to 2/nm
c. It is involved in the generation of ionic gradients between the cytoplasm and the
external environment
d. It serves as a sensor of signals from the extra-cellular environment
Ans-28: (b) It possesses a thickness of 1 to 2 nm
Ref.:Read the text below
Sol :
Cell membranes range in thickness from 7 to 10nm.
A cell membrane surrounds all eukaryotic cells and sub serves a number of essential
functions.
The plasma membrane forms a boundary to the external environment and contains a
large variety of receptors that function as ligands for hormones, growth factors, cytokines,
and other extra cellular factors.
Its a selective barrier that controls the entry and exit of substances to and from the
cell.
The plasma membrane is involved in both energy dependent (active) and energy
independent (passive) transport and the generation of ionic gradients between inside and
outside of the cell.
The plasma membrane and the internal membranes of the cell such as those
surrounding organelles like the nucleus and mitochondria are similar in their morphologic
appearance under electron microscopy.
The organelles membranes are usually thinner and differ in overall biochemical
composition from that of the plasma membrane.

Q-29 Closure of the aortic valve occurs at the onset of which phase of the cardiac cycle?
a. Isovolumetric contraction
b. Rapid ejection
c. Protodiastole
d. Isovolumetric relaxation
Ans-29: (d) Isovolumetric relaxation
Ref.:Read the text below
www.damsdelhi.com

Email: info@damsdelhi.com

DAMS CBT 2016 SET - 1

Sol :
Closure of the semilunar valves (aortic and pulmonic valves) marks the beginning of the
isovolumetric relaxation phase of the cardiac cycle.
During this brief period (approximately 0.06s), the ventricles are closed and myocardial
relaxation, which began during protodiastole, continues.
Intraventricular pressure falls rapidly, although ventricular volume changes little.
When intraventricular pressure falls below atrial pressure, the mitral and tricuspic
valves open and rapid filling of the ventricles begins.

Q-30 An increase in plasma PTH level would lead to an increase in which of the
following?
a. Renal synthesis of calcitriol
b. Renal proximal tubular reabsorption of Ca+2
c. The number of active osteoblasts
d. Plasma inorganic phosphate concentration
Ans-30: (a) Renal synthesis of calcitriol
Ref.:Read the text below
Sol :
The major renal effect of parathyroid hormone (PTH) is stimulation of proximal tubular
1 hydroxylase, an enzyme that converts calcidiol to calcitriol.
PTH affects renal Ca+2 reabsorption in two ways: it reduces Ca+2 reabsorptionin the
proximal tubule and increases Ca+2 reabsorption in the distal tubule.
The net effect is an increase in tubular Ca+2 reabsorption.
This renal effect of PTH can be lessened by PTHs effect on bone.
PTH increases plasma [Ca+2] by promoting bone resorption, which causes an increase in
the renal filtered load of Ca+2 and, thus, hypercalciuria.
Also, PTH inhibits the proximal reabsorption of phosphate, thereby favoring phosphate
excretion.
PTH acts on bone by dissolving the non readily exchangeable calcium phosphate pool
known as stable bone.
PTH activates the osteoclast cells that cause osteolysis by their high content of
lysosomal enzymes. PTH also stimulates osteocytes, which are bone bound osteoblasts
that mediate osteocytic osteolysis.

Q-31 Which of these are not a gluconeogenic substrate :


a. Oxaloacetate
b. Pyruvate
c. Acetyl-CoA
d. Lactate
Ans-31: (c) Acetyl-CoA
Ref: Harper : illustrated biochemistry 30/e p185
Sol:
Acetyl Co A can not form glucose, rest all other substrates given in other options are
capable of gluconeogenesis.
www.damsdelhi.com

Email: info@damsdelhi.com

DAMS CBT 2016 SET - 1

Following are gluconeogenic substrates


Glycerol
Fumarate
Propionic acid
Glucogenic amino acid
Pyruvate
Lactate

Q-32 All of the following are required in PCR except?


a. Deoxyribonucleotides
b. Thermostable enzyme
c. Dideoxyribonucleotides
d. DNA Template
Ans-32: (c) Dideoxyribonucleotides
Ref: Read the text below
Sol:
A basic PCR set up requires several components and reagents. These components include:
DNA template that contains the DNA region (target) to be amplified.
Two primers that are complementary to the 3' (three prime) ends of each of thesense
and anti-sense strand of the DNA target.
Taq polymerase or another DNA polymerase with a temperature optimum at around
70 C.
Deoxynucleoside triphosphates (dNTPs, sometimes called "deoxynucleotide
triphosphates"; nucleotides containing triphosphate groups), the building-blocks from which
the DNA polymerase synthesizes a new DNA strand.
Buffer solution, providing a suitable chemical environment for optimum activity and
stability of the DNA polymerase.
2+
2+
Divalent cations, magnesium or manganese ions; generally Mg is used, but Mn can
be utilized for PCR-mediated DNA mutagenesis, as higher Mn2+concentration increases the
error rate during DNA synthesis
Monovalent cation potassium ions.

Q-33 Second messenger for smooth muscle relaxation mediated by NO is :


a. cGMP
b. cAMP
c. Ca2+
d. Magnesium
Ans-33: (a) cGMP
Ref: Harper: Illustrated biochemistry 30/e p660
Sol:

www.damsdelhi.com

Email: info@damsdelhi.com

DAMS CBT 2016 SET - 1

NO is paracrine in action as after getting produced in endothelial cell from arginine,it is


diffused in underlying smooth muscle where it is activating guanylyl cyclase which
converts GTP to cGMP.
This cGMP phosphorylates muscle protein which undergo relaxation.

Q-34 Which of these amino acid does not enter the krebs cycle by forming
Acetyl-CoA via pyruvate :
a. Tyrosine
b. Hydroxyproline
c. Glycine
d. Alanine
Ans-34: (a) Tyrosine
Ref: Harper: Illustrated biochemistery 30/e p165
Sol:
List of amino acid which,can form acetyl co A
Hydroxyproline
Cysteine
Glycine
Serine
Threonine
Alanine will form pyruvate
Tyrosine and phenylalanine will form fumarate
Valine, isoleucine, methionine will form Succinyl co A

Q-35 Aminoacyl tRNA synthetases must be capable of recognizing which of the


following?
a. A specific tRNA and a specific amino acid
b. A specific rRNA and a specific amino acid
c. A specific tRNA and the 40S ribosomal subunit
d. A specific amino acid and the 40S ribosomal subunit
Ans-35: (a) A specific tRNA and a specific amino acid
Ref: Read the text below
Sol:
There are 20 isoenzymes for aminoacyl tRNA Synthetase enzyme.
The respective isoenzyme for charging of tRNA is selected by D arm.

Q-36 The lactose operon is negatively controlled by the lactose repressor and positively
controlled by which of the following?
a. Increase concentration of glucose and cyclic AMP
b. Decrease concentration of glucose and cAMP
c. Increase concentration of glucose, decrease concentration of cAMP

www.damsdelhi.com

Email: info@damsdelhi.com

DAMS CBT 2016 SET - 1

d. Decrease concentration of glucose, Increase concentration of cAMP


Ans-36: (d) Decrease concentration of glucose, Increase concentration of cAMP
Ref: Read the text below
Sol:
The lactose operon is negatively controlled by the lactose repressor and positively
controlled by decreasing concentration of glucose and increase concentration of cAMP

Q-37 The metabolic adaptation in alcoholic is all of the following except?


a. Lactic acidosis
b. High NAD+ level
c. Accumulation of fat in the liver
d. Decreased uric acid excretion
Ans-37: (b) High NAD+ level
Ref: Read the text below
Sol:
Oxidation of ethanol by alcohol dehydrogenase leads to excess production of NADH.
The increased (NADH/NAD+) ratio also causes increased (lactate/pyruvate) , resulting in
hyperlacticacidemia, which decreases excretion of uric acid, aggravating gout.
Increased NADH/NAD ratio will prevent fatty acid oxidation and hence fatty acid will
tend to deposit in hepatocyte leading to fatty liver.

Q-38 In the synthesis of collagen, the hydroxylation of proline and lysine occurs in which
of the following?
a. Golgi apparatus
b. Secretory vesicles
c. Rough endoplasmic reticulum
d. Smooth endoplasmic reticulum
Ans-38: (c) Rough endoplasmic reticulum
Ref: Alberts, pp 10981100. Junqueira, pp 106110
Sol:
Prolyl and lysyl hydroxylase are the two enzymes that carry out hydroxylation of
proline and lysine.
The process is both co- and posttranslational and, therefore, occurs during, or more
often after, the amino acids are inserted into nascent collagen polypeptide chains in the
RER.
Hydroxyproline, which constitutes 10% of collagen, is often used to determine the
collagen content of various tissues.
Hydroxylation of proline stabilizes the triple helix through interchain hydrogen bonds,
and hydroxylation of lysine is critical for the cross-linking stage of collagen assembly.
Q-39 A child to emergency with accidental ingestion of Cyanide . It blocks citric
acid cycle by blocking :
a. Acetyl-CoA Production

www.damsdelhi.com

Email: info@damsdelhi.com

DAMS CBT 2016 SET - 1

b. Aconitase
c. NAD+
d. Citrate
Ans-39: (c) NAD+
Ref: Illustrated Biochemistry 30/ep132
Sol:
Cyanide inhibits electron transport chain and hence stops ATP synthesis, depleting NAD.
In lack of NAD , TCA cycle does not run as NAD is important coenzyme for three
dehydrogenases in this cycle.
1. Isocitrate dehydrogenase
2. Alpha keto glutarate dehydrogenase
3. Malate dehydrogenase

Q-40 Calcium absorption is increased in the GUT by :


a. 1,25 dihydroxycholecalciferol
b. Intrinsic factor
c. Calcitonin
d. PTH
Ans-40: (a) 1,25 dihydroxycholecalciferol
Ref: Read the text below
Sol:
In liver, Vitamin D is hydroxylated at the 25th position, and in kidney further
hydroxylation is affected at the 1st position to produce dihydroxy-cholecalciferol or
calcitriol.
The calcitriol induces a carrier protein in the intestinal mucosa, which increases the
absorption of calcium. Hence blood calcium level tends to be elevated.
Vitamin D is acting independently on bone. Vitamin D increases the number and
activity of osteoblasts, the bone forming cells.
Secretion of alkaline phosphatase by osteoblasts is increased by vitamin D

Q-41 LDL lipoprotein is associated with :


a. Apoprotein A2
b. Apoprotein B100
c. Apoprotein C1
d. Apoprotein E
Ans-41: (b) Apoprotein B100
Ref: Read the text below
Sol:

Low-density lipoprotein (LDL) is one of the five major groups of lipoproteins, which in
order of size, largest to smallest, are chylomicrons, VLDL, IDL, LDL, and HDL, that enable
transport of various lipid in plasma.
www.damsdelhi.com

Email: info@damsdelhi.com

DAMS CBT 2016 SET - 1

Each native LDL particle contains a single apolipoprotein B-100 molecule (Apo B-100, a
protein that has 4536 amino acid residues and a mass of 514 kDa.

Q-42 Which among the following is a feature of non competitive inhibition?


a. Increased Vmax
b. Decreased Vmax
c. Increased Km
d. Decreased Km
Ans-42: (b) Decreased Vmax
Ref: Read the text below
Sol:
A non competitive inhibitor has no effect on Km. but decrease Vmax
Vmax : maximum velocity
Km : the substrate concentration at which the enzyme attains half of the Vmax

Acting on
Structure of inhibitor
Inhibition is
Excess substrate
Km
Vmax
Significance

Competitive inhibition
Active site
Substrate analogue
Reversible
Inhibition relieved
Increased
No change
Drug action

Non competitive inhibition


May or may not
Unrelated
Generally irreversible
No effect
No change
Decreased
Toxicological

Q-43 Insulin Dependent glucose Transport occurs in all except


a. Skeletal muscle
b. Heart
c. Liver
d. Adipocyte
Ans-43: (c) Liver
Ref: Read the text below
Sol:
GLUT 4 is the transporter of glucose which is dependent on insulin for its action.
This type of GLUT is distributed at skeletal muscle, heart muscle and adipose cell.

Q-44 Selenocysteine is derived from


a. Serine
b. Cysteine
c. Anethionine
d. Alanine
Ans-44: (a) Serine
Ref: Read the text below

www.damsdelhi.com

Email: info@damsdelhi.com

DAMS CBT 2016 SET - 1

Sol:
Serine is hydroxyl group containing amino acid. Its oxygen is replaced by selenium as to
give rise to selenocysteine.

Q-45 Proximal Histidine is


a. E7
b. E8
c. F7
d. F8
Ans-45: (d) F8
Ref: Read the text below
Sol:
F8 is proximal histidine and E7 is distal histidine.
Q-46 A 10 year old boy, Pappu, died of acute rheumatic fever. All the following can be
expected at autopsy except.
a. Ashoff nodules
b. Rupture of chordate tendinae
c. Mc Callum patch
d. Fibrinous pericarditis
Ans-46: (b) Rupture of chordate tendinae
Ref.:Robbins - 566
Sol :
Pathological features of Rheumatic carditis
Acute Rheumatic carditis
There is diffuse inflammation and aschoff bodies in any of the three layers of heart
Pericardium Fibrinous or serofibrinous pericarditis also known as bread & butter
pericarditis.
Pericarditis generally resolve without sequelae.
Myocardium Diffuse non-specific myocarditis with Aschoff bodies.
Endocardium Small warty projection, i.e. verucae along the line of closure on the
valvular leaflets (especially on left side mitral valve.)

Q-47 Characteristic feature of Rheumatic carditis is


a. Pericarditis
b. Endocarditis
c. Myocarditis
d. Pancarditis
Ans-47: (d) Pancarditis
Ref.:Robbins - 566
Sol :
During Acute Rheumatic fever, diffuse inflammation and Aschoff bodies may be found
in any of the three layers of heart-pericardium, myocardium or endocardium- Pancarditis.

www.damsdelhi.com

Email: info@damsdelhi.com

DAMS CBT 2016 SET - 1

Q-48 The commonest mode of inheritance of von willebrands diasease


a. Codominant
b. Autosomal recessive
c. Autosomal dominant
d. X linked recessive
Ans-48: (c) Autosomal dominant
Ref.:Robbins - 671
Sol :
Von willebrand disease is of following types
Type I (most common 70%) Autosomal dominant, with reduced quantity of vWF.
Type II (25%) Autosomal dominant, with qualitative defect of vWF.
Type III Autosomal recessive, with reduced quantity of vWf.
Q-49 The presence of small sized platelets on the peripheral smear is characteristic ofa. Idiopathic thrombocytopenic purpura.
b. Bernard soulier syndrome
c. Disseminated intravascular coagulation
d. Wiskott Aldrich syndrome
Ans-49: (d) Wiskott Aldrich syndrome
Ref.:Robbins 8th/e p. 263
Sol :
Characteristics of Wiskott Aldrich syndrome
Severe eczema
Thrombocytopenia
Recurrent infections
The platelets are not only reduced in number but more characteristically are small.
Other characteristic abnormality seen in Wiskott Aldrich syndrome is that peripheral T
cells appear bald and devoid of microvilli.

Q-50 A female patient nandini presents with upper respiratory tract infection. Two days
after. She develops hematuria. Probable diagnosis is
a. IgA nephropathy
b. Wegners granulomatosis
c. Henoch sholein purpura
d. Post streptococcal glomerulonephritis
Ans-50: (a) IgA nephropathy
Ref.:Robbins - 931
Sol :
DIFFERENTIATING FEATURES B/W IGA NEPHROPATHY AND POSTSTREPTOCOCCAL G.N.
IgA nephropathy
Poststreptococcal glomerulonephritis
Onset of hematuria 2-4 days after URTI Onset of hematuria 7-21 days following
Serum complement C3 normal
URTI and skin infection

www.damsdelhi.com

Email: info@damsdelhi.com

DAMS CBT 2016 SET - 1

Recurrence common
No elevation of antibody titre

Serum complement C3 decreased


Recurrence rate, one attack confers life
long immunity
Elevation of antistreptococcal antibody
titre

Q-51 Which component or HBV causes glomerulonephritis


a. Hbe Ag.
b. HBc Ag.
c. HBs Ag.
d. Anti HBs Ag antibody
Ans-51: (a) Hbe Ag.
Ref.:Harrison - 1938
Sol :
Immune complex mediated tissue damage is the cause of glomerulonephritis in the
host.
Immune complex consistsHbs Ag and antiHBs.
Hbs Ag is a structural compent HBV, while Anti Hbs Ag is formed by the host
So, Hbs Ag is the component of HBV that is responsible for glomerulonephritis.

Q-52 All of the following are associated with low complement levels except
a. Lupus nephritis
b. Mesanogio capillary glomerulonephritis
c. Diarrhea associated hemolytic uremic syndrome
d. Glomerulonephritis related to bacterial endocarditis
Ans-52: (c) Diarrhea associated hemolytic uremic syndrome
Ref.:Read the text below
Sol :
Lupus Nephritis, post streptococcal glomerulonephritis and membrano proliferative
glomerulonephritis are all associated with low complement levels

Q-53 The Electron Microscopy is virtually diagnostic in renal biopsy study of


a. Goodpastures syndrome
b. Churg-Strauss Syndrome
c. Alport syndrome
d. Wegners granulomatosis
Ans-53: (c) Alport syndrome
Ref.:Robbins - 932
Sol :
Alports syndrome presents with characteristic histological features on electron
microscopy.
Such changes may present in other diseases, but are most pronounced and widespread
in Alports syndrome.
www.damsdelhi.com

Email: info@damsdelhi.com

DAMS CBT 2016 SET - 1

Features of Alports syndrome on electron microscopy :


Alports patients early in their disease typically have diffuse thining of the basement
membrane which thickens over time into multilamellous surrounding lucent areas that
often contain granules of varying density.
The characteristic electron electron microscopic finding of fully developed disease is
that glomerular basement membrane shows irregular foci of thickening alternating with
attenuation (thinning) with pronounced splitting and lamination of lamina densa often
with distinctive basket weave appearance.

Q-54 Renal papillary necrosis is almost always associated with one of the following
conditions
a. Diabetes mellitus
b. Analgesic nephropathy
c. Chronic pyelonephritis
d. Post streptococcal GN
Ans-54: (a) Diabetes mellitus
Ref.:Dorlands Illustrated Medical Dictonary 28th / e.p. 1104
Sol :
Renal papillary necrosis, an accompaniment of acute pyelonephritis is most often seen
in diabetics and is characterized by necrosis of renal papillae of one or both kidneys with
shaped demarcation between necrotic and living tissue .

Q-55 All are features of hemolytic uremic syndrome, except


a. Hyperkalemia
b. Anaemia
c. Renal microthrombi
d. Neuro psychiatric disturbances
Ans-55: (d) Neuro psychiatric disturbances
Ref.:Robbins - 669
Sol :
Hyperkalemia is seen in hemolytic urenic syndrome as a result of renal failure.
About neuropsychiatric manifestations
Neurological manifestations are used to distinguish between Hemolytic uremic
syndrome and Thrombotic Thrombocytopenic Purpura.
H.U.S. is distinguished from T.T.P by the absence of neurological symptoms and the
prominence of acute renal failure.

Q-56 The following carcinomas are associated with DIC except


a. Stomach
b. Pancreas
c. Prostate
d. Breast
Ans-56: (d) Breast
www.damsdelhi.com

Email: info@damsdelhi.com

DAMS CBT 2016 SET - 1

Ref.:Robbins - 673
Sol :
Cancers associated with DIC
Ca pancreas Ca lung - Acute promyelocytic leukemia
Ca protstate - Ca Stomach
Note : Breast Ca. can also cause DIC, but amongst the given options, it is least
frequently associated with DIC.

Q-57 Although more than 400 blood groups have been identified, the ABO blood group
system remains the most important in clinical medicine because
a. It was the first blood group system to be discovered
b. It has four different blood groups A, B, AB, O (H)
c. ABO(H) antigens are present in most body tissues and fluids
d. ABO(H) antibodies are invariably present in plasma when persons RBC lacks the
corresponding antigen.
Ans-57: (d) ABO(H) antibodies are invariably present in plasma when persons RBC lacks
the corresponding antigen.
Ref.:Robbins -263
Sol :
The basic difference between ABO blood group system and other blood group systems
(such as Rh, Kell, Duffy, MNSs blood groups), which makes the ABO group so important is
that.
Preformed ABO antibodies (agglutinins) are present in persons serum when his RBCs
lack the corresponding antigen (ie. Anti B antibody would be present in a person of person
of type A and type O blood groups. These two blood groups do not have the B antigen on
the RBCs)
This is not the case with other blood groups.
Preformed antibodies are not present. They are formed only after an exposure to the
antigen, for example Rh negative person do not have anti- Rh antibodies, these antibodies
are formed only after an exposure to Rh positive blood (by a blood transfusion, i.e. this
makes the first blood transfusion safe even if mismatched.

Q-58 Most common type of hodgkins lymphoma in India isa. Nodular sclerosing
b. Lymphocyte predominance
c. Mixed cellularity
d. Lymphocyte depletion
Ans-58 (c) Mixed cellularity
Ref.:Robbins 8th/e p. 928
Sol :
While nodular sclerosis is the most common subtype in the most common subtype in
the developed countries, the mixed cellularity type is most common in developing
countries.

www.damsdelhi.com

Email: info@damsdelhi.com

DAMS CBT 2016 SET - 1

Q-59 Histologic examination of the lesion in stomach reveal fat-laden cells, likely causes is
a. Lymphoma
b. Postgastrectomy
c. Signet-cell carcinoma stomach
d. Atrophic gastritis
Ans-59 : (b) Postgastrectomy
Ref.:Read the text below
Sol :
Lipid laden macrophages (foam cells) are seen in xanthomatosis of stomach.
Xanthomatosis of stomach
It is characterized by collection of lipid laden macrophages (foam cells) forming plaques
or nodules.
Gastric xanthomatasis is more common in patients with :
Gastritis
Gastric ulcer
Deudenogastric reflux ofter gastric surgery.

Q-60 The gene involved in GIST


a. C-KIT
b. BRAC-1
c. p53
d. BRAC-2
Ans-60: (a) C-KIT
Ref.:Robbins - 789
Sol :
Pathogenesis of GISTs
Approximately 75% to 80% of all GISTs have oncogenic, gain-of-function mutations of
gene encoding the gyrosine kinase c-KIT.
Approximately 8% of GISTs have mutations that activate a related tyrosine kinase,
platelet derived growth factor receptor (PDGFRA).
Consitutively active c-KIT or PDGRA receptor tyrosine kinases activate RAS and
P13K/AKT pathways and thereby promote tumor cell proliferation.

Q-61 In a patient with acute leukemia, immunophenotype pattern is CD 19+ve, CD 10+ve,


CD33+ve, CD 13+ve, He may probably have
a. Biphenotypic leukemia
b. ALL
c. AML M2
d. AML- M0
Ans-61: (a) Biphenotypic leukemia
Ref.:Robbins - 263
Sol :

www.damsdelhi.com

Email: info@damsdelhi.com

DAMS CBT 2016 SET - 1

These type of acute leukemia with immunophenotypic features of more than one cell
lineage are referred to as acute leukemias of ambiguous lineage in new classification
system proposed by world health organization
Biphenotypic leukemia is a subtype of leukemia of ambiguous lineage in which the
malignant cells population express markers of two different lineage- most commonly
myeloid and either B or T lymphoid lineage.

Q-62 Which of the following is not compatible with a diagnosis of juvenile


myelomonocytic leukemia
a. Peripheral blood monocytosis, more than 1 x 109/L
b. Increased hemoglobin F levels for age
c. Presence of ber/abl fusion gene
d. GM-CSF hypersensitivity of myeloid progenitors in vitro
Ans-62: (c) Presence of ber/abl fusion gene
Ref.:Robbins - 1968
Sol :
Juvenile myelomonocytic leukemia is a clonal proliferation of hematopoietic stem cells
that typically affects children younger than 2 years of age.
Patients with this disease do not experess bcr/abl fusion gene and the also do not have
Philadelphia chromosome which is seen in 90% of cases of adult CML.
Q-63 Lymphoepithelial lesions in GIT are seen in all except:
a. IPSD (Immune proliferative small instestinal Disease)
b. MALTOMA
c. Celiac disease
d. Adenocarcinoma intestine
Ans-63: (d) Adenocarcinoma intestine
Ref.:Robbins - 795
Sol :
Lymphoepithelial lesions are seen in lymphomas of GIT
When extreme numbers of atypical lymphoid cells populate the superficial or
glandular epithelium , it is called lymphoepithelial lesions.
Various intestinal lymphomas are.
MALT lymphoma (MALTOMA)
Immunoproliferative small intestinal disease (IPSID) also referred to as Mediterranean
lymphoma.
Intestinal T cell lymphoma
Predisposing factors to lymphoma are :
Chronic gastritis caused by H. Pylori
Organ transplantation with immunosuppression.
Celiac disease.
Congenital immunodeficiency.

Q-64 All of the following conditions may show Mallory Hyaline changes
www.damsdelhi.com

except

Email: info@damsdelhi.com

DAMS CBT 2016 SET - 1

a. Wilson disease
b. Indian childhood cirrhosis
c. Primary biliary cirrhosis
d. Hepatitis E
Ans-64: (d) Hepatitis E
Ref.:Robbins - 858
Sol :
Mallory hyaline (Mallory bodies)
Mallory bodies are eosinophilic intracytoplasmic inclusion bodies which are composed
predominantly of keratin intermediate filaments.
They are also known as alcoholic hyaline.
These inclusions are characteristic but not specific feature of alcoholic liver disease, as
they are also seen in other conditions.

Q-65 Raised iron content is not found in which organ in Hemochromatosis


a. Heart
b. Skin
c. Testis
d. Pituitary
Ans-65 : (c) Testis
Ref.:Robbins - 862
Sol :
In hemochromatosis hypogonadism is caused by impairment of hypothalamic pituitary
function and not due to deposition of Iron in the Testis.
Q-66 Kerley B lines are seen in which part of chest X-ray:
a. Upper portion
b. Mid portion
c. Peripheral lower zone
d. Upper peripheral zone
Ans-66: (d) Peripheral lower zone
Ref: Sutton 7th/e Pg 289
Sol:
Kerley B lines seen in the peripheral part of the base of the lung (lower zone) are
diagnostic of pulmonary venous hypertension.
These are seen as horizontal lines at the lung base.

Q-67 A 25 years old young male presents with sudden onset dyspnea and chest pain. His
chest is hyper resonant to percussion on the left side. The X ray picture is shown below

www.damsdelhi.com

Email: info@damsdelhi.com

DAMS CBT 2016 SET - 1

The diagnosis is:


a. Consolidation
b. Pneumothorax
c. Flailchest
d. Pleural effusion
Ans67: (b) Pneumothorax
Ref: Sutton 5th ed ; Grainger and Allison 5th ed chapter 13
Sol:
Sudden onsetdyspnoea and chest pain along with a hyperresonantnot( due to air) to
percussion on the left side is suggestive of a diagnoses of Pneumothorax amongst the given
options.
Also the X-ray picture depicting a raised hemidiaphragm on the left side and a
deepsulcus sign(increasedlucent area over the costophrenicrecess) goes in favour of a
diagnosis of Pneumothorax.

Q-68 A 50 years old male presented with fatigue, recurrent upper respiratory tract
infections and headache. On investigating he was found to have anemia, hypercalcemia
and elevated urine protein. The image of his skull X ray is given in

www.damsdelhi.com

Email: info@damsdelhi.com

DAMS CBT 2016 SET - 1

What is the diagnosis:


a. Chronic fatigue syndrome
b. Multiple myeloma
c. Hemolytic anemia
d. Brain metastasis
Ans-68: (b) Multiple myeloma
Ref: Sutton 7thed; Grainger and Allison 5thed chapter 74
Sol:
The clinical picture suggests a diagnosis of Multiple myeloma and the characteristic
skull
X-ray picture depicting punched out lytic lesions also favours a diagnosis of Multiple
myeloma.
Note: Skull X ray picture depicting lytic lesions with bevelled edges is seen in patients
with Eosinophillic granuloma.

Q-69 The most sensitive imaging modality for diagnosis of ureteric stones in a patient
with acute colic is
a. X-ray
b. USG
www.damsdelhi.com

Email: info@damsdelhi.com

DAMS CBT 2016 SET - 1

c. NCCT Abdomen
d. CECT Abdomen
Ans-69: (c) NCCT Abdomen
Ref: Grainger and Allisons Diagnostic Radiology 5th/e, Radiology in surgical practice 1st/e
Pg 220
Sol:
Best investigation for the diagnosis of renal and ureteric stones is NCCT(Non Contrast
Spiral CT) Abdomen.

Q-70 Riglers sign is seen in


a. Hydatid cyst
b. Pneumoperitoneum
c. Intussusception
d. Crohns disease
Ans-70: (b) Pneumoperitoneum
Ref: Suttons Radiology, 7th ed., p. 676
Sol:
Riglers sign is seen in pneumoperitoneum.
Visualization of the outer as well as inner wall of a loop of bowel is seen in
pneumoperitoneum and this is called Riglers sign.

Q-71 Comb sign is seen in:


a. Crohn disease
b. Ulcerative colitis
c. Pseudomyxoma peritonei
d. Typhoid fever
Ans-71: (a) Crohn disease
Ref: Read the text below
Sol:
"Comb sign is seen in Crohn disease.
It refers to the hypervascular appearance of the mesentry in active Crohn disease.
Fibro-fatty proliferation and peri-vascular inflammatory infiltrates are seen outlining
the intestinal border.
It can be seen in both CT and MRI but is not pathognomic of Crohn disease."

Q-72 Step ladder pattern on X-ray is seen in:


a. Perforation peritonitis
b. Ulcerative colitis
c. Intestinal obstruction
d. Meckels diverticulum
Ans-72: (c) Intestinal obstruction
Ref: Read the text below

www.damsdelhi.com

Email: info@damsdelhi.com

DAMS CBT 2016 SET - 1

Sol:
"Step ladder pattern on X-ray is seen in intestinal obstruction.
It occurs due to multiple air-fluid level.
This occurs because beyond the obstruction there is no gas and only fluid with
distension of the intestines.
Other sign seen in intestinal obstruction is string of beads sign."
Suttons Radiology, 7th ed, p. 1539

Q-73 The trefoil which is used in international radiation warning symbol has three blades
and the angle between each blade should be
a. 45
b. 50
c. 60
d. 70
Ans-73: (c) 60
Ref: Read the text below
Sol:
The international radiation symbol (also known as trefoil) first appeared in 1946, at the
University of California, Berkeley Radiation Laboratory.

At the time, it was rendered as magenta, and was set on a blue background.
The modern version used in the US is magenta against a yellow background, and it is
drawn with a central circle of radius R, an internal radius of 1.5R and an external radius of
5R for the blades, which are separated from each other by 60.
The trefoil is black in the international version, which is also acceptable in the US.

Q-74 Linear acceleratoris for


a. Electrons and xrays
b. Gamma and protons
c. Electrons and protons
d. Neutrons and xray
Ans-74: (a) Electrons and xrays
Ref: Read the text below
Sol:
LINAC produces both xrays and electrons.
A linear accelerator (LINAC) is the device most commonly used for external beam
radiation treatments for patients with cancer.
It delivers high-energy x-rays to the region of the patient's tumor.

www.damsdelhi.com

Email: info@damsdelhi.com

DAMS CBT 2016 SET - 1

Q-75 Bear claw sign on CT1


a. HCC
b. Hepatic laceration
c. RCC
d. Pancreatic laceration
Ans-75: (b) Hepatic laceration
Ref: Read the text below
Sol:

Q-76 According to ICD 10 classification by WHO 2016 version how many chapters are
there in ICD X?
a. 20
b. 21
c. 22
d. 23
Ans-76: (c) 22
Ref: Read the text below
Sol:
International Classification of disease is produced by WHO for national and international
use
It has been revised over years and the latest revision ICD 10, came into effect on January 1,
1993 is arranged in 21 chapters. The 2016 modification has made it into 22 chapters.

www.damsdelhi.com

Email: info@damsdelhi.com

DAMS CBT 2016 SET - 1

I Certain infectious and parasitic diseases


II Neoplasms
III Diseases of the blood and blood-forming organs and
certain disorders involving the immune mechanism
IV Endocrine, nutritional and metabolic diseases
V Mental and behavioural disorders
VI Diseases of the nervous system
VII Diseases of the eye and adnexa
VIII Diseases of the ear and mastoid process
IX Diseases of the circulatory system
X Diseases of the respiratory system
XI Diseases of the digestive system
XII Diseases of the skin and subcutaneous tissue
XIII Diseases of the musculoskeletal system and connective
tissue
XIV Diseases of the genitourinary system
XV Pregnancy, childbirth and the puerperium
XVI Certain conditions originating in the perinatal period
XVII Congenital malformations, deformations and
chromosomal abnormalities
XVIII Symptoms, signs and abnormal clinical and laboratory
findings, not elsewhere classified
XIX Injury, poisoning and certain other consequences of
external causes
XX External causes of morbidity and mortality
XXI Factors influencing health status and contact with health
services
XXII Codes for special purposes

www.damsdelhi.com

Email: info@damsdelhi.com

DAMS CBT 2016 SET - 1

Q77 As per ESI rules 2016, dependant gets:


a. A job
b. 100% wage
c. 90% wage
d. 80% wage
Ans-77: (c) 90% wage
Ref: Read the text below
Sol:
New ESI Guidelines 2016
Heads

Benefit Facility

Medical

Full medical care

Sickness

50% of daily wage is payable for 91 days

Extended sickness

Temporary disablement

80% of daily wage payable for 2 years (730 days) for a set of
34 diseases
Full daily wage up to 7 days for vasectomy and up to 14 days
for tubectomy
Full daily wages
up to 12 weeks for confinement
up to 6 weeks for miscarriage or MTP
up to 4 weeks for sickness arising out of pregnancy,
confinement, premature birth
90% of daily wage till recovery

Permanent disablement

90% of daily wage

Dependant

Pension at 90% of wages

Funeral expenses

Cash amounting to 10000/-

Enhanced sickness
Maternity

Rehabilitation

Q-78 WHO definition for impairment is?


a. Difficulty to carry out activities that is considered normal for the age
b. Any loss or abnormality of psychological, physiological or anatomical structure or
function
c. Deviation from normal anatomy
d. Any disability that prevents the fulfillment of a role that is normal

www.damsdelhi.com

Email: info@damsdelhi.com

DAMS CBT 2016 SET - 1

Ans-78: (b) Any loss or abnormality of psychological, physiological or anatomical structure


or function
Ref: Read the text below
Sol:
Any loss or abnormality of psychological, physiological or anatomical structure or function
is impairment
Disability - Any restriction or lack of ability to perform an activity in the manner or
within the range considered normal for a human being is disability
Handicap - A disadvantage for a given individual resulting from an impairment or a
disability that limits or prevents the fulfillment of a role that is normal (depending on age,
sex and social and cultural factors) for that individual

Q-79 Which is the correct formula to calculate specific death rates due to TB for males
among the following?
a. No. of deaths in males during a calendar year x 1000 / No. of TB deaths
b. No. of deaths due to TB among males during a calendar year x 1000 / No. of death
among malesduring the calendar year
c. No. of deaths due to TB among males during a calendar year x 1000 / Mid year
population of males
d. No. of deaths due to TB among males during a calendar year x 1000 / No. of diagnosed
cases of TB during the calendar year
Ans-79: (c) No. of deaths due to TB among males during a calendar year x 1000 / Mid year
population of males
Ref: Read the text below
Sol:
Specific death rate = No of deaths due to the disease during a calendar year x 1000 /
Midyear population

Q-80 Identify the picture used for health promotion:

www.damsdelhi.com

Email: info@damsdelhi.com

DAMS CBT 2016 SET - 1

a. Flip chart
b. Flannel chart
c. Pictogram
d. Flash cards
Ans-80: (a) Flip chart
Ref: Read the text below
Sol:

Q-81 Which of the following is true regarding Attack rate?


a. An incidence rate used only when population is exposed to risk for a limited period of
time
b. Indicates the number of exposed persons develops disease within the incubation period
c. No. of all current cases of suspected disease at a a specific period of time
d. Average number of spells of disease in each calendar year
Ans-81: (a) An incidence rate used only when population is exposed to risk for a limited
period of time
Ref: Read the text below
Sol:
An incidence rate used only when population is exposed to risk for a limited period of
time such as during an epidemic
Usually expressed in %
It reflects the extent of epidemic
Attack rate = No. of new cases of a specified disease during a specified time interval x
100 / Total population at risk during the same interval

Q-82
following

www.damsdelhi.com

To calculate Incidence rate during the


year the cases to be considered are?

Email: info@damsdelhi.com

DAMS CBT 2016 SET - 1

a. 1, 2, 3, 4,6,7,8
b. 2,3,4,7
c. Case 8 only
d. Case 2 & Case 4
Ans-82: (b) 2,3,4,7
Ref: Read the text below
Sol:
Incidence rate is defined as the nmber of new cases occurring in a defined population
during a specified period of time
It is not influenced by duration of the disese

Q-83 In case of disaster epidemiology mitigation means?


a. Lowering the impact
b. Preventing the impact
c. Early warning
d. Triage
Ans-83: (a) Lowering the impact
Ref: Read the text below
Sol:
Mitigation is the effort to reduce loss of life and property by lessening the impact of
disasters. In order for mitigation to be effective we need to take action nowbefore the
next disasterto reduce human and financial consequences later (analyzing risk, reducing
risk, and insuring against risk). It is important to know that disasters can happen at any time
and any place and if we are not prepared, consequences can be fatal.
Effective mitigation requires that we all understand local risks, address the hard choices,
and invest in long-term community well-being. Without mitigation actions, we jeopardize
our safety, financial security and self-reliance.

Disasters can happen at anytime and anyplace; their human and financial consequences
are hard to predict.

The number of disasters each year is increasing but only 50% of events trigger Federal
assistance.

FEMA's mitigation programs help reduce the impact of eventsand our dependence on
taxpayers and the Treasury for disaster relief.

www.damsdelhi.com

Email: info@damsdelhi.com

DAMS CBT 2016 SET - 1

Q-84 If cut off for a screening test in serum cholesterol level is increased, what will
happen to the number of False positives:
a. Increase
b. Decrease
c. Increase exponentially
d. No change
Ans-84: (b) Decrease
Ref: Read the text below
Sol:
The following terms are fundamental to understanding the utility of clinical tests:
1. True positive: the patient has the disease and the test is positive.
2. False positive: the patient does not have the disease but the test is positive.
3. True negative: the patient does not have the disease and the test is negative
4. False negative: the patient has the disease but the test is negative.
If the cut-off point is raised, there are fewer false positives but more false negativesthe
test is highly specific but not very sensitive. Similarly, if the cut-off point is low, there are
fewer false negatives but more false positivesthe test is highly sensitive but not very
specific.

Q-85 The number of diseases eradicated till date are:


a. 0
b. 1
c. 2
d. 3
Ans-85: (b) 1
Ref: Read the text below
Sol:
So far only one disease, smallpox, has been eradicated by vaccines, saving
approximately 5 million lives annually.
Polio could be next.
Over 80% of the world's children are now being immunized against the polio virus, and
the annual number of cases has been cut from 400,000 in 1980 to 90,000 in the mid-1990s

Q-86 Current Tobacco pack warnings by MOHFW, GoI are:


a. 40% on one side
b. 85% on both sides
c. Plain packaging
d. Any of the above will do
Ans-86: (b) 85% on both sides
Ref: Read the text below
Sol:
On October 15, 2014, Indias Ministry of Health and Family Welfare (MOHFW)
announced new rules called the Cigarettes and Other Tobacco Products (Packaging and
www.damsdelhi.com

Email: info@damsdelhi.com

DAMS CBT 2016 SET - 1

Labelling) Amendment Rules, 20146 to amend the previous 2008 Packaging and Labelling
Rules7 .
These new rules increased the size of PHWs on smoked and smokeless tobacco
packages, effective April 1, 2015.
The new warnings would cover 85% of the principal display areas (both sides at the top
edge) of tobacco packages, of which 60% would consist of specified pictorial warnings and
25% would consist of the text warning.
This represents a substantial increase from the current size of PHWs, which cover 40%
of the principal display area of the front panel of the packages

Q-87 Which of the following is an experimental study?


a. Ecological
b. Cross-sectional
c. Case control
d. Randomized control trial
Ans-87: (d) Randomized control trial
Ref: Read the text below
Sol:

Epidemiological
Methods

Experimental /
Intervention
Studies

Observational
Studies

Descriptive
Studies

Ecological /
Correlational

Analytical
Studies

Cross sectional
/ Prevalence

Randomized
Controlled
Trials

Case control /
Case Reference

Field Trials

Community
Trials

Cohort

Q-88 The primary role of ophthalmic examination in PHC is entrusted to?


a. Medical officer
b. Ophthalmologist
c. Optometrist
d. Any one of these
Ans-88: (c) Optometrist

www.damsdelhi.com

Email: info@damsdelhi.com

DAMS CBT 2016 SET - 1

Ref: Read the text below


Sol:
Under NPCB optometrist is posted at level of PHC
Q-89 Which of the following is a live attenuated vaccine?
a. Plague
b. Pertussis
c. Varicella
d. Rabies
Ans-89: (c) Varicella
Ref: Read the text below
Sol:
Plague Pertussis and Rabies are killed whole organism
Important vaccines currently in use include
Live
Killed
Toxoid /
Polysacharide Glycoconjugate
attenuate whole
Protein
d
organis
m
BCG
Typhoid Diphtheri Pneumococcus Hib Pneumococcus
OPV
Cholera
a
Meningococcu MeningococcusACW
Measles
Plague
Tetanus
s
Y
Mumps
Pertussis anthrax
Hib
Rubella
IPV
Typhoid
Yellow
Rabies
fever
JE
Typhoid
TBE
Varicella
HAV
Rotavirus

Recombinan
t

HBV
HPV
Lyme disease

Q-90 Brand name of condoms available under the national program is:
a. Nirodh
b. ASHA
c. ASHOK
d. Chatri
Ans-90: (b) ASHA
Ref: Read the text below
Sol:
Recently the brand name of condoms available under the national program has been
changed from Nirodh to ASHA

Q-91 MALA D contains


a. Levonorgestral 0.15 mg and ethinyl estradiol 0.03 mg
b. Norgestral 0.15 mg and ethinyl estradiol 0.03mg

www.damsdelhi.com

Email: info@damsdelhi.com

DAMS CBT 2016 SET - 1

c. Levonorgestral 0.25 mg and ethinyl estradiol 0.15 mg


d. Norgestral 0.25 mg and ethinyl estradiol 0.15 mg
Ans-91: (a) Levonorgestral 0.15 mg and ethinyl estradiol 0.03 mg
Ref: Read the text below
Sol:
MALA N and MALA D are the low dose oral pill made available by Government of India.
MALA N and MALA D contains Levonorgestral 0.15 mg and ethinyl estradiol 0.03 mg
MALA N is supplied free of cost through all PHCs, Urban family welfare centrs etc

Q-92 An oriya boy married a haryanvi girl. This will lead to:
a. Mores
b. Acculturation
c. Folkways
d. Nothing will happen
Ans-92: (b) Acculturation
Ref: Read the text below
Sol:
Acculturation refers to the processes that occur when different individuals or groups of
people meet and exchange aspects of their culture.
Due to advances in transportation, communication, and technology, there has been a
significant increase in the interactions among different cultures.
As a result, understanding acculturation, and learning how to accomplish it effectively,
has become vital to the future of the world.

Q-93 Infant mortality rate (per thousand live births) in of India as per the recent SRS is:
a. 40
b. 39
c. 38
d. 37
Ans-93: (a) 39
Ref: Read the text below
Sol:

Q-94 As per WHO classification mild MR is


a. IQ 50 70
b. IQ 55 75
c. IQ 50 65
d. IQ 45 35
Ans-94: (a) IQ 50 70
Ref: Read the text below
Sol:
Mild MR IQ 50 70

www.damsdelhi.com

Email: info@damsdelhi.com

DAMS CBT 2016 SET - 1

Moderate MR IQ 35 49
Severe MR IQ 20 34
Profound MR IQ under 20

Q-95 Cut off for CD 4 count for treatment of HIV and TB confected currently in India is:
a. 200
b. 350
c. 500
d. Treat the moment either is diagnosed
Ans-95: (c) 500
Ref: Read the text below
Sol:
WHO guidelines4 recommend that, among HIV-positive individuals, ART should be
initiated for:

anyone age five years or older with a CD4 count of 500 cells/mm3

all children over age five regardless of CD4 count if they are in WHO clinical stage 3 or 4
or have active TB disease

all children under five years old, including infants

all pregnant and all breastfeeding women

all serodiscordant couples

all people coinfected with TB

all people coinfected with hepatitis B who present with chronic liver disease.
Q-96 Rajesh, a 7-month-old child, presents with failure of gaining weight and noisy
breathing which becomes worse when the child cries. Laryngoscopy shows a reddish mass
in subglottis. Which of the following is not a part of treatment modality in this patient?
a.
Radiation
b.
Steroids
c.
Tracheostomy
d.
Carbon dioxide laser treatment
Ans-96: (a) Radiation
Ref.:Read the text below
Sol :

The diagnosis is subglottic hemangioma.

Radiotherapy is not a treatment option for the same while CO2 laser, intraregional
steroids and tracheostomy are tried.
Q97 A 38-year-old gentleman reports of right sided deafness for the past 2 years; on
testing with a 512 Hz tuning fork, the Rinne test without masking is negative on the right
side and positive on the left side. On doing weber, the tone is perceived louder on the left
ear. The patient most likely has :
a.
Right Conductive deafness
b.
Right Sensorineural deafness
c.
Left Sensorineural deafness

www.damsdelhi.com

Email: info@damsdelhi.com

DAMS CBT 2016 SET - 1

d.
Left conductive deafness
Ans-97: (b) Right Sensorineural deafness
Ref.: Read the text below
Sol :

For right sided deafness, if Weber is lateralized to left that indicates sensorineural
hearing loss.

Due to transcranial transmission to the opposite normal cochlea in severe


sensorineural deafness, namely false negative Rinne on the right as explained above.

Patient has a severe sensorineural deafness on the right side.

Q98 Which is the correct statement regarding facial nerve injury in temporal bone
fractures?
a.
Commoner in transverse fractures
b.
Commoner in longitudinal fractures
c.
Facial palsy is of immediate onset
d.
It is always associated with CSF otorrhea
Ans-98: (a) Commoner in transverse fractures
Ref.:Read the text below
Sol :

Facial palsy is commoner with transverse fractures (in>50%). It may be of immediate


onset in transverse fractures since the fracture line may run across the facial canal leading
to trans-section.

But facial palsy is rare (<20%) in longitudinal fractures since the fracture line usually
runs above the external and middle ear and parallel to the facial canal. When present, it is
delayed in longitudinal fractures since it is usually due to edema of the nerve.

CSF otorrhea need not manifest in transverse fractures since the tympanic membrane
is intact while it is a usually accompaniment of longitudinal fractures.

Q99 Sade classification is related with retraction of pars tensa,Grade 2 includes


a.
Mild retraction
b.
Retracted TM touching incus or stapes
c.
TM touching promontory but moves on seigelisation
d.
TM touching promontory, does not move
Ans-99: (b) Retracted TM touching incus or stapes
Ref:Read the text below
Sol:
Sade classification of retraction of pars tensa
Grade 1 mild retraction
Grade 2severe retractionretracted TM touching incus or stapes
Grade 3Atelectatic TMTM touching promontory but moves on seigelisation
Grade 4---Adhesive TM--TM touching promontory, does not move on seigelisationos
classification of pars flaccida
Toss classification of pars flaccida
Stage I: Pars flaccida is dimpled and is more retracted than normal. It is not adherent to
www.damsdelhi.com

Email: info@damsdelhi.com

DAMS CBT 2016 SET - 1

the malleus.
Stage II: In this stage the retraction pocket is adherent to the handle of malleus. The full
extent of the retraction pocket can be clearly seen.
Stage III: In this stage part of the retraction pocket may be hidden. There may also be
associated erosion of the outer attic wall (scutum).
Stage IV: In this stage there is definite severe erosion of the outer attic wall. The extent of
the retraction pocket cannot be clearly seen as most of it are hidden from the view.

Q-100 In Otosclerosis, carharts notch dips at :


a.
1000 Hz in air conduction
b.
1000 Hz in bone conduction
c.
2000 Hz in air conduction
d.
2000 Hz in bone conduction
Ans-100: (d) 2000 Hz in bone conduction
Ref.:Read the text below
Sol :

The dip in audiogram occurs for bone conduction curve in otosclerosis and is
maximum of about 15 dB at 2000 Hz. It may involve other frequenceies from 500 Hz to
4000 Hz to a lesser extent (5-10 dB).

The Carharts notch though demonstrable in bone conduction curve does not signify
sensorineural hearing loss and disappears after a successful stapedectomy.

Q-101 A 55-year-old female presents with tinnitus, dizziness, and h/o progressive deafness.
Which of the following is not a differential diagnosis for this?
a.
Acoustic neuroma
b.
Endolymphatic hydrops
c.
Meningioma
d.
Histiocytosis X
Ans-101: (d) Histiocytosis X
Ref.:Read the text below
Sol :

All except histiocytosis present with triad of vertigo, tinnitus, and deafness.

Histiocytosis X belong to the group of disorders collectively termed inflammatory


reticuloendotheliosis characterized by multiple osteolytic lesions involving skull, temporal
bone, long bones, ribs, and vertebrae.

There is generalized lymphadenopathy, hepatosplenomegaly, and in severe cases


involvement of the bone marrow.

Involvement of temporal bone leads to features mimicking complicated CSOM like


otorrhea, mastoiditis, facial palsy, and labyrinthitis.

Q-102 Inverted papilloma is characterized by all except :


a.
Also called as schneiderian papilloma
b.
Seen more often in females
www.damsdelhi.com

Email: info@damsdelhi.com

DAMS CBT 2016 SET - 1

c.
Presents with epistaxis and nasal obstruction
d.
Originates from lateral wall of nose
Ans-102: (b) Seen more often in females
Ref: Dhingra 216
Sol :

Inverted papilloma is a transitional cell papilloma also called Schineiderian


papilloma/Ringertz Tumor.

It is the most common benign neoplasm of the nose and paranasal sinuses.

It presents in the middle age (40-70 yrs.) as soft friable mass resembling polyp.

More common in males (male : female = 5 : 1).

Arises almost exclusively from the lateral wall of the nose and only occasionally from
the septum.

It is always unilateral.

Patients mainly complain of nasal obstruction, rhinorrhea & unilateral epistaxis,

There is coincidental malignancy else where in the upper respiratory tract in about 4%
of the cases and malignant transformation of the tumor occurs in about 8% cases.

Treatment is adequate local excision. If it arises in maxillary sinus, then a radical


antrostomy is carried out. If it arises in the ethmoidal sinus, an external ethmodiectomy is
done. If it arises from nose, treatment is wide surgical excision by lateral rhinotomy

Has a tendency to recur even after removal.

Q-103 About nasal syphilis which of the following is untrue :


a.
Perforation occurs in septum
b.
Saddle nose deformity may occur
c.
In new born, it presents as snuffles
d.
Secondary syphilis is the common association
Ans-103: (d) Secondary syphilis is the common association
Ref: Dhingra 173
Sol :

Nasal syphilis may be :

Acquired :
Primary e.g. chancre in vestibule

Secondary e.g. simple rhinitis, crusting and fissuring

Tertiary e.g. Gumma

Congenital :

Early (first 3 months) : Presenting as snuffles, purulent nasal discharge, fissuring


excoriation.

Late (around puberty) : Gumma in septum and other stigmatas.

Saddle nose deformity occurs due to collapse of nasal bridge.

Atrophic rhinitis is a complication.

Teritary syphilis is a common association : primary and secondary is rare association in


nasal syphilitic.

Septal perforation occurs in bony part in case of syphilis.

Q-104 Which of the following is not typical feature of Menieres disease?


www.damsdelhi.com

Email: info@damsdelhi.com

DAMS CBT 2016 SET - 1

a.
Sensorineural deafness
b.
Pulsatile tinnitus
c.
Vertigo
d.
Fluctuating deafness
Ans-104: (b) Pulsatile tinnitus
Ref.:Read the text below
Sol :

Episodic vertigo and fluctuant sensorineural hearing loss are features of Menieres
disease.

It is associated with tinnitus, which is usually a low pitched roaring or hissing sound.
Pulsatile tinnitus is not a feature of Menieres disease and occurs with conditions like
arteriovenous malformations of the temporal bone, glomus tumor, etc.

Q-105 Tympanoplasty is most commonly performed for tympanic membrane perforations


greater than:
a.
10 20% of the size of the entire tympanic membrane
b.
20 30% of the size of the entire tympanic membrane
c.
30 40% of the size of the entire tympanic membrane
d.
40 50% of the size of the entire tympanic membrane
Ans-105: (a) 10 20% of the size of the entire tympanic membr
Ref:Read the text below
Sol:
Tympanoplasty is most commonly performed for tympanic membrane perforations
greater than 10 20% of the size of the entire tympanic membrane.
Tympanoplasty alone or type 1 tympanoplasty denotes that the surgery is confined to the
eardrum alone, and no manipulation of the middle ear bones (ossicles) occurs.
Tympanoplasty that includes manipulation of the ossicles is defined as tympanoplasty
with ossicular reconstruction, or tympanoossiculoplasty.
Overall, there is approximately 95 98% success rate in closing tympanic membrane
perforations with tympanoplasty.
There are three ways to approach the tympanic membrane
Transcanal Approach(Most Common)
The transcanal (meaning, through the ear canal) approach is performed exclusively through
the ear canal except for a small incision behind the ear, which is used to obtain the graft
material.
Postauricular Approach
Endaural approach

Q-106 A 27 year old female presents with tender subcutaneous nodules on lower legs.
Which of the following is least likely to be the etiology?

www.damsdelhi.com

Email: info@damsdelhi.com

DAMS CBT 2016 SET - 1

Use of oral contraceptives


a. Streptococcal infection
b. Sarcoidosis
c. Giant cell arteritis
Ans-106: (d) Giant cell arteritis
Ref: Read the text below
Sol:
The diagnosis is Erythema nodosum.
It can occur as a result of
Infections(eg:Streptococcal,Fungal,Mycobacterial,Yersinial)
Drugs(eg:Sulfadrugs,Penicillins,Oral contraceptives)
Sarcoidosis.

Q-107 Shagreen patch is aa. Neurofibroma


b. Neuroma
c. Collagenoma
d. Angiofibroma
Ans-107: (c) Collagenoma
Ref: Read the text below
Sol:
The shagreen patch is most commonly found on the back or flank area; it is an
irregularly shaped, slightly raised, or textured skin lesion. The lesion is found in 20 to 30%
of patients with tuberous sclerosis complex.
Hypomelanotic macules (ash leaf spots) are found in over 90% of patients with tuberous
sclerosis complex. They are usually present at birth but are often difficult to see in the
newborn without an ultraviolet light.
Adenoma Sebaceum (Facial angiofibromas) occur in about three fourths of patients but
often appear several years after the diagnosis has been established by other means. These
lesions typically become apparent during the preschool years as a few small red papules on

www.damsdelhi.com

Email: info@damsdelhi.com

DAMS CBT 2016 SET - 1

the malar region; they gradually become larger and more numerous, sometimes extending
down the nasolabial folds or onto the chin. Angiofibromas contain both vascular and
connective tissue elements. Although facial angiofibromas are a strong indication of
tuberous sclerosis complex when found with other manifestations, these lesions also occur
in individuals with multiple endocrine neoplasia type I and thus are not pathognomonic for
either condition
Ungual fibromas are nodular or fleshy lesions that arise adjacent to or from underneath
the nails. Ungual fibromas are seen in about 20% of unselected patients with tuberous
sclerosis complex and are more likely to be found in adolescents or adults than in younger
children.

Q-108 Slit skin smear is positive if


a. >104 bacilli /gm of tissue
b. >103 bacilli /gm of tissue
c. >102 bacilli /gm of tissue
d. >105 bacilli /gm of tissue
Ans-108: (a) >104 bacilli /gm of tissue
Ref: Read the text below
Sol:
Slit skin smear is positive if >104 bacilli /gm of tissue

Q-109 A suspected case of pemphigus vulgaris is best investigated with


a. DIF of a fresh blister
b. DIF of a perilesional sample
c. Light microscopy of an old blister
d. Electron microscopy
Ans-109: (b) DIF of a perilesional sample
Ref: Read the text below
Sol:
Immuno bullous diseases are best diagnosed by demonstrating the antibodies bound in
the skin sample by direct immunofluorescence (DIF).
For DIF samples are best taken from perilesional skin.

Q-110 A child had a scratch on its cheek, who complained of fever the next day.Within
days the infection spread to ear.Most probable diagnosis is:
a. Cellulitis
b. Erysipelas
c. Ecthyma
d. Erythrasma
Ans-110: (b) Erysipelas
Ref: Read the text below
Sol:

www.damsdelhi.com

Email: info@damsdelhi.com

DAMS CBT 2016 SET - 1

Milans ear sign: ear is involved in erysipelas and not in cellulitis as it does not have
deep dermis.
Q-111 The histopathology image shown below points to a diagnosis of ?

a. Pemphigus vulgaris
b. Pemphigus foliaceus
c. Psoriasis
d. Lichen planus
Ans-111: (d) Lichen planus
Ref: Read the text below
Sol:
The characteristic histological changes seen in Lichen planus are;
i)Basal epidermal cell degeneration causing saw tooth appearance of rete pegs and
eosinophiliccytoid bodies(Civatte bodies)
ii)Epidermal thickening,especially of granular cell layer
iii)Sub-epidermal lichenoid band due to deposition of lymphocytes and histiocytes.
iv)Max Joseph spaces or small areas of artifactual separation between the epidermis and
the dermis

Q-112 A 3 months old male infant developed otitis media for which he was given a course
of Co-trimoxazole. A few days later, he developed extensive peeling of the skin , there
were no mucosal lesions and the baby was not toxic,Diagnosis:
a. Toxic epidermal necrolysis
b. Staphylococcal scalded skin syndrome
c. Steven Johnson syndrome
d. Infantile pemphigus
Ans-112: (b) Staphylococcal scalded skin syndrome

www.damsdelhi.com

Email: info@damsdelhi.com

DAMS CBT 2016 SET - 1

Ref: Read the text below


Sol:
Staphylococcal scalded skin syndrome.
It is caused by Staphylococcus aureus of phage group II (here otitis dermatitis) which
produce epidermolytic toxins.
These toxins cleave desmoglein 1 and produce widespread peeling of stratum
corneum.
Since desmoglein 1 is not expressed in mucosae, there is no mucosal involvement.
Steven Johnson syndrome and Toxic epidermal necrolysis are drug reactions with
purpuric lesions, severe mucosal involvement and deeper level of split with full thickness
peeling of epidermis.
In SSSS patient is generally well but in SJS and TEN patient is quite ill.

Q-113 According to new research, which is most strongly associated with pathogenesis of
psoriasis
a. Th1
b. Th2
c. Th17
d. Th18
Ans-113: (c) Th17
Ref: Read the text below
Sol:
Psoriasis earlier believed to be mediated by predominantly TH1 cells (CD 8+); but now
TH17 appears to be most important pathogenic subset of T- cells induced by IL-23 from
psoriatic plaques.
IL-8 may be important for attracting neutrophils in the plaques (esp in pustular
psoriasis).

Q-114 Islands of sparing (nappes claires) are a classical feature of


a. Pityriasis versicolor
b. Pityriasis rosea
c. Pityriasis rubra pilaris
d. Pityriasis alba
Ans-114: (c) Pityriasis rubra pilaris
Ref: Read the text below
Sol:
Palmoplantar keratoderma (orange-red, waxy, hyperkeratosis of palms and soles
known as PRP sandal)
Red (salmon colored ) scaly plaques with islands of sparing (nappes claires) i.e
normal skin within erythematous plaques
Perifollicular, hyperkeratotic, rough, horny papules over trunk and especially over
digits simulating a nutmeg grater appearance and feel.

www.damsdelhi.com

Email: info@damsdelhi.com

DAMS CBT 2016 SET - 1

Q-115 Which of the following is not associated with atopic dermatitis


a. Posterior subcapsular cataract
b. Anterior subcapsular cataract
c. Keratoconus
d. Hertoghes sign
Ans-115: (a) Posterior subcapsular cataract
Ref: Read the text below
Sol:
Many eye/periorbital changes are associated with atopic dermatitis including:
Anteriorsubcapsular cataract
Keratoconus, periorbital pallor (headlight sign)
Periorbital darkness (allergic shiners)
Lateral madarosis (Hertoghes sign)
Accessory and deep infraorbital lines (Dennies lines/ Dennie Morgan folds).

Q-116 What is the most common presentation of Ashermans syndrome


a. Infertility
b. Menstrual irregularities
c. Pain
d. Abortions
Ans-116: (b) Menstrual irregularities
Ref: Read the text below
Sol :
The most common presentation is menstrual irregularities while the single most
common complaint is infertility
Among menstrual irregularities the most common is amenorrhoea followed by
hypomenorrhoea

Q-117 A primigravida comes for first antenatal visit at 12 weeks she is B negative and her
husband blood group is B positive and ICT is negative which of the following g is the best
step in her management
a. Give her 300mcg Inj Anti D
b. Give her 50 mcg of Inj Anti D
c. Repeat her ICT at 28 weeks
d. Nothing needs to be done as this is her first pregnancy
Ans-117: (c) Repeat her ICT at 28 weeks
Ref: Read the text below
Sol :
She needs a repeat ICT at 28 weeks and if that is also negative then we will give her
300mcg of anti D

www.damsdelhi.com

Email: info@damsdelhi.com

DAMS CBT 2016 SET - 1

Q-118 All are used as post coital contraceptives except


a. High dose estrogens
b. Cu T
c. LNG
d. Ulipristal
Ans-118: (a) High dose estrogens
Ref: Read the text below
Sol :
Estrogen alone are not used as emergency contraceptives although OCP are used as
emergency contraception
Ulipristal is the most effective hormonal EC
While LNG tab is the most commonly used
CuT is the most effective among all

Q-119 A lady G2P1L1 comes to you at 20 weeks POG with a history of first child suffering
from downs which if the following is the management for her
a. Nothing to be done as she is beyond 20 weeks
b. CVS
c. Amniocentesis
d. Cordocentesis
Ans-119: (c) Amniocentesis
Ref: Read the text below
Sol :
Amniocentesis is the safest procedure to obtain sample for karyotyping and it can be
done anytime after 15 weeks.
The cells used for karyotyping are amniocytes and fibroblast.
Early amniocentesis before 15 weeks is no longer done

Q-120 Which of the following dose of Misoprostol is incorrectly matched


a. Prophylaxis of PPH 600mcg
b. MTP 800mcg
c. Labor induction at term 25mcg
d. Treatment of PPH 800mcg
Ans-120: (b) MTP 800mcg
Ref: Read the text below
Sol :
The dose of Misoprostol used for out patient medical abortion upto 7 weeks is 400
mcg
The other drug used with it is mifepristone in a dose of 200mg

Q-121 Congenital diaphragmatic hernia can be diagnosed by USG as early as :


a. 11 weeks

www.damsdelhi.com

Email: info@damsdelhi.com

DAMS CBT 2016 SET - 1

b. 15 weeks
c. 24 weeks
d. 30 weeks
Ans-121: (a) 11 weeks
Ref: Read the text below
Sol :
The CDH can be diagnosed as early as 11 weeks although most commonly it is
diagnosed at around 15 weeks
The anomaly that can be earliest diagnosed on USG is anencephaly as early as 10
weeks

Q-122 All are true about thyroid physiology in pregnancy except


a. Increase in TBG
b. Increase in free T4 T3
c. Goitre
d. Decrease in TSH
Ans-122: (c) Goitre
Ref: Read the text below
Sol :
Goitre is never physiological it is pathological finding in pregnancy
Because of stimulation of the thyroid gland by HCG both total and free t4 and t3
increase.

Q-123 A pregnant mother presents with history of delivery of a previous child with
CongenitalAdrenal Hyperplasia (CAH). The best management protocol for the current
pregnancy is :
a. To start prednisolone after establishing whether fetus is affected by Chorionic Villous
Sampling
b. To start dexamethasone as soon as pregnancy is confirmed
c. To start dexamethasone after determining sex of the fetus by Karyotyping
d. To start prednisolone after determining sex of the fetus with USG
Ans-123: (b) To start dexamethasone as soon as pregnancy is confirmed
Ref: Read the text below
Sol :
"The best management protocol for the current pregnancy in a pregnant mother
presents with history of delivery of a previous child with Congenital Adrenal Hyperplasiais is
to start dexamethasone as soon as pregnancy is confirmed.
To achieve best results treatment must be started as soon as pregnancy is recognized
and no later than 9 weeks.
Dexamethasone (20mg /kg) is the agent of choice for fetal adrenal suppression.
The diagnosis of CAH in a sibling places the fetus of current pregnancy at risk of CAH."

www.damsdelhi.com

Email: info@damsdelhi.com

DAMS CBT 2016 SET - 1

Q-124 Which of the following dietary supplements is recommended for a pregnant lady on
Heparin:
a. Folic acid
b. Copper
c. Zinc
d. Calcium
Ans-124: (d) Calcium
Ref: Read the text below
Sol :
Calcium dietry supplements is recommended for a pregnant lady on Heparin.
Heparin does not cross blood brain barrier or placenta and it is the anticoagulant of
choice in pregnancy.
However use of heparin may be associated with transient hypocalcemia and hence
calcium supplementation during pregnancy may be advised

Q-125 Risk factors for Placenta Accreta include all of the following except:
a. Previous LSCS scar
b. Previous curettage
c. Previous myomectomy
d. Previous placenta praevia
Ans-125: (d) Previous placenta praevia
Ref: Read the text below
Sol :
Placenta Praevia in the present pregnancy (not previous placenta previa) constitutes a risk
factor for placenta Accreta.
Note:
Risk Factors for placenta Accreta include placenta previa (Present pregnancy), Previous
curettage, Multiparity, previous cesarean section, previous myomectomy and advanced
maternal age ( 35 years)

Q-126 Which of the following conditions is not diagnosed by Chorionic Villous Biopsy?
a. Neural tube defects
b. Downs syndrome
c. Phenylketonuria
d. Sickle cell anemia
Ans-126: (a) Neural tube defects
Ref: Read the text below
Sol :
"Neural tube defects are not associated with any of the above and are not detected by
chronic villous biopsy.
Chorionic villous sampling may be used to detect disorders with cytogenetic,
biochemical (genetic) or molecular disorder."

www.damsdelhi.com

Email: info@damsdelhi.com

DAMS CBT 2016 SET - 1

Q-127 What is the ideal treatment for a 55 yr female with Simple Hyperplasia of
endometrium with Atypia?
a. Simple hysterectomy
b. Medroxy progesterone Acetate (MPA)
c. Levonorgesterol (LNG)
d. Mirena
Ans-127: (a) Simple hysterectomy
Ref: Read the text below
Sol :
The ideal treatment for a 55 yr female with Simple Hyperplasia of endometrium with
Atypia is simple hysterectomy.
Endometrial hyperplasia is regarded as a precursor for endometrial carcinoma.
Presence of cytological atypia and post menopausal age (55 years) suggest increased
risk of progression to carcinoma, and hence this patient should be treated with
hysterectomy."

Q-128 Salpingitis /Endosalpingitisis best confirmed by:


a. Endometrial biopsy
b. Laparoscopy + hysteroscopy
c. Hysterosalpingography
d. Sonosalpingography
Ans-128: (b) Laparoscopy + hysteroscopy
Ref: Read the text below
Sol :
"Salpingitis /Endosalpingitis is best confirmed by by Hysteroscopy + Laparoscopy.
The diagnosis of salpingitis is best confirmed by Laparoscopy.
Hysterectomy may also provide confirmatory evidence of salpingitis.
Hence Hysteroscopy + Laparoscopy is the single best answer of choice."

Q-129 The first sign of puberty in girls is:


a. Breast budding
b. Peak height velocity
c. Menarche
d. Pubic and axillary hair growth
Ans-129: (a) Breast budding
Ref: Read the text below
Sol :
First sign of puberty in females is growth spurt while the first visible sign of puberty is
Thelarche which is appearance of breast buds.
Growth spurt is beginning of skeletal growth which is not synonymous to peak height
velocity

www.damsdelhi.com

Email: info@damsdelhi.com

DAMS CBT 2016 SET - 1

Q-130 In a case of complete Mllerian duct Aplasia all the following are likely to be absent
except :
a. Uterus
b. Fallopian tubes
c. Vagina
d. Ovaries
Ans-130: (d) Ovaries
Ref: Read the text below
Sol :
Ovaries are gonads and they develop from genital ridge and not from Mllerian ducts
while all other structures in the question are derived from Mllerian ducts.
Also the karyotype of the patient will be normal 46 XX.

Q-131 Vaginal Delivery may be recommended in:


a. Monochorionic Monoamniotic Twins
b. First twin vertex, second twin breech
c. First twin extended breech, second twin vertex
d. First twin me to posterior , and second twin breech
Ans-131: (b) First twin vertex, second twin breech
Ref: Read the text below
Sol :
The outcome of twin pregnancy depends on lie of first twin, if the first twin is
longitudinal vaginal delivery can be done.
While if the first twin is non longitudinal then CS is done.
If the first twin is breech and second vertex again CS is preferred because of risk of
twin entrapment
Monism optic twins have a risk of cord entanglement and they are delivered by CS

Q-132 Minimum duration between onset of symptoms and death is seen in:
a. APH
b. SEPTECEMIA
c. PPH
d. Obstructed labor
Ans-132: (c) PPH
Ref: Read the text below
Sol :
The most common cause of death in obstetric patient is PPH and the progression is
also rapid.
The most common cause of shock after delivery is PPH

Q-133 The most effective contraceptive method recommended in lactating mothers is

www.damsdelhi.com

Email: info@damsdelhi.com

DAMS CBT 2016 SET - 1

a. Barrier method
b. Progesterone only pill
c. Oral contraceptive Pills
d. Lactational Amenorrhea
Ans-133: (b) Progesterone only pill
Ref: Read the text below
Sol :
Progesterone only pills (POPs) are the most effective contraceptives during lactation
amongst the options provided.
POPs are more effective than Barrier methods and lactational amenorrhea (alone).

Q-134 Which of the following is true about obstructive azoospermia


a. Increase FSH and increaseLH
b. increase FSH and Normal LH
c. Increase LH, Normal FSH
d. Normal FSH, Normal LH
Ans-134: (d) Normal FSH, Normal LH
Ref: Read the text below
Sol :
The hormone levels including Lh FSH and testosterone are normal in a patient with
obstructive azoospermia.
While in non obstructive the testosterone level is low and LH bad FSH are high.
The single most important hormone for evaluation is serum testosterone

Q-135 Which method of contraception is shown?

www.damsdelhi.com

Email: info@damsdelhi.com

DAMS CBT 2016 SET - 1

A) Falope ring
B) Hysteroscopic
C) Modified pomeroy
D) Laparoscopic
Ans-135: (b) Hysteroscopic
Q-136 Which of the following is not true about physiological changes in pregnancy?
a. Renal blood flow and GFR decreases by 50%
b. Plasma volume increases by 40 %
c. Plasma protein concentration is decreased by 10%
d. Respiratory rate and Vital capacity remains unchanged.
Ans-136: (a) Renal blood flow and GFR decreases by 50%
Ref: Read the text below
Sol :
Renal blood flow and GFR increases by about 50% during pregnancy as a result of
which serum creatinine levels decrease and excretion of glucose increases during
pregnancy.
Glucosuria is a normal physiological finding seen in pregnancy.

Q-137 Ideal time for termination of pregnancy in cases with Intrahepatic cholestasis is:
a. 32 weeks
b. 34 weeks
c. 36 weeks
www.damsdelhi.com

Email: info@damsdelhi.com

DAMS CBT 2016 SET - 1

d. 38 weeks
Ans-137: (d) 38 weeks
Ref: Read the text below
Sol :
Ideal time for termination of pregnancy in cases with intrahepatic cholestasis is 37
weeks.
Most common clinical manifestation of intrahepatic cholestasis in pregnancy is
Pruritis.
Drug of choice for Intrahepatic cholestasis of pregnancy is Ursodeoxycholic acid.

Q-138 Minimum hCG level at which gestational sac can be detected by tranvaginal
songoraphy is
a. 1500
b. 1000
c. 2000
d. 2500
Ans-138: (b) 1000
Ref: Read the text below
Sol :
Express critical Titer of hcg for TVS is 1500IU/l but the minimum value at which it can
pick up sac may be 1000Iu/l
The critical Titer for transabdominal ultrasound is 6500 Iu/l

Q-139 The most common chromosomal anomaly to be associated with abortions is


a. Trisomy 16
b. Trisomy 21
c. Monosomy X
d. Trisomy 18
Ans-139: (c) Monosomy X
Ref: Read the text below
Sol :
The single most common cause of abortions is monosomy X followed by trisomy 16

Q-140 Embryo reduction of multiple pregnancyis done at


a. 8-10 weeks
b. 11-13 weeks
c. 13-15 weeks
d. 16-18 weeks
Ans-140: (b) 11-13 weeks
Ref: Read the text below

www.damsdelhi.com

Email: info@damsdelhi.com

DAMS CBT 2016 SET - 1

Sol :
Embryo reduction of multiple pregnancy ( i.e. converting a triplet or quadruplet
pregnancy to twin pregnancy ) is done at 11-13 weeks period of gestation
Embryo reduction is done using injection potassium chloride in the fetal heart under
USG guidance.

Q-141 All of the following are true about pancreatic pseudocyst except?
a. It is a collection of amylase rich fluid
b. Develops within 2 weeks after the onset of acute pancreatitis.
c. Pseudocyst fluid has a low CEA level
d. It may resolve spontaneously
Ans-141: (b) Develops within 2 weeks after the onset of acute pancreatitis.
Ref: Read the text below
Sol:
Pseudocyst of pancreas
Pseudocyst of pancreas is a collection of amylase-rich fluid enclosed in a wall of fibrous
or granulation tissue.
It typically arise following an attack of acute pancreatitis, but can develop in chronic
pancreatitis or after pancreatic trauma.
Formation of a pseudocyst requires 4 weeks or more from the onset of acute
pancreatitis.
Fluid from a pseudocyst typically has a low CEA level, and levels above 400 ng/mL are
suggestive of a mucinous neoplasm.
Pseudocysts will resolve spontaneously in most instances.
Therapeutic interventions are advised only if the pseudocyst causes symptoms, if
complications develop or a distinction has to be made between a pseudocyst and a
tumour.

Q-142 Which of the following is not an intramural cause of intestinal obstruction?


a. Stricture
b. Hernia
c. Intususseption
d. Volvulus
Ans-142: (b) Hernia
Ref: Read the text below
Sol:

www.damsdelhi.com

Email: info@damsdelhi.com

DAMS CBT 2016 SET - 1

Q-143 Which one of the following plastic surgeon won the noble prize for his work?
a. Harold Gillies
b. T.P.Kilner
c. Paul Tessier
d. Joseph Murray
Ans-143: (d) Joseph Murray
Ref: Read the text below
Sol:
It was Joseph Murray who performed the first successful renal transplant in
identical twins in 1955, for which he was conferred the Noble prize.
The other options are of leaders and icons of plastic surgery based on their
individual contributions to the specialty.

Q-144 The TRAM (transverse rectus abdominis musculocutaneous) flap has


revolutionized breast reconstruction. All of the following can be considered advantages
and indications of the free TRAM flap over pedicled TRAM except which one of the
following?
a. Reduced abdominal dissection and muscle sacrifice.
b. History of heavy cigarette use (10 packs/ years smoking)
c. High incidence of flap failure
d. Avoidance of disturbance of medial inframammary fold
Ans-144: (c) High incidence of flap failure
Ref: Read the text below
Sol:
Both the pedicled and free TRAM procedures may be indicated for patients desiring
breast reconstruction.
Patients in high-risk categories,such as those with a history of heavy cigarette use(>10
pack/years smoking) and those who are overweight or obese, are more suitable for free
than for pedicle TRAM reconstruction.

www.damsdelhi.com

Email: info@damsdelhi.com

DAMS CBT 2016 SET - 1

Advocates for the free TRAM cite its advantages of reduced abdominal dissection and
muscle sacrifice , enhanced flap vascularity , ease of flap inset, and avoidance of
disturbance of the medical inframammary fold.
Requirement of microvasular skills and significantly higher risk of total flap failure
are disadvantages of the free TRAM flap technique.

Q-145 The deformity crows feet is located at which one of the following region?
a. Feet of the crow
b. Perioral region
c. Perineal region
d. Periobicular region
Ans-145: (d) Periobicular region
Ref: Read the text below
Sol:
Crows feet are lateral periorbital wrinkles due to muscle hyperactivity combined
with senile degeneration of the overlying skin.
They can be treated temporarily using Botox injection therapy but definitive
treatment may require suborbicularis midfacial elevation.
The rest of the options are incorrect.

Q-146 Which of the following nerve is not in relation to submandibular gland?


a. Lingual nerve
b. Hypoglossal nerve
c. Marginal mandibular branch of facial nerve
d. Vagus nerve
Ans-146: (d) Vagus nerve
Ref: Read the text below
Sol:
Important anatomical relationships of the submandibular glands
Lingual nerve
Hypoglossal nerve
Anterior facial vein
Facial artery
Marginal mandibular branch of the facial nerve

Q-147 Trauma and injury severity score (TRISS) includes


a. GCS + BP +BR
b. Revised trauma score+ injury severity score + age
c. Revised trauma score + injury severity score + GCS
d. Revised trauma score+ GCS+BP
Ans-147: (b) Revised trauma score+ injury severity score + age
Ref: Read the text below

www.damsdelhi.com

Email: info@damsdelhi.com

DAMS CBT 2016 SET - 1

Sol:
Trauma and injury severity score (TRISS) combines the revised Trauma score (RTS) and
injury severity score (ISS) with the Age of patient together with the method of injury
(Blunt or penetrating).
Trauma and injury severity score (TRISS)
The TRISS is a combined scoring system used to predict the outcome following major
trauma
It combines the anatomic derangements (ISS) and physiological derangements (RTS)
together with patients Age and Mechanism of Injury (Blunt or penetrating) to predict
survival after trauma.
..
TRISS (Trauma and injury Severity Score).
Injury severity Score (ISS)
Revised Trauma Score (RTS)
Age
Mechanism of injury (Blunt/penetrating)
.
The TRISS scoring system provides a rough predicator medicating whether morality in a
given patient is expected to be greater or less than 50 percent.

Q-148 Which of the following is not an extra intestinal manifestation of acute colitis?
a. Arthritis
b. Cirrhosis
c. Pleural effusion
d. Uveitis
Ans-148: (c) Pleural effusion
Ref: Read the text below
Sol:
Extraintestinal manifestations of ulcerative colitis.
Arthritis occurs in around 15 per cent of patients.
Sacroiliitis and ankylosing spondylitis are 20 times more common in patients with UC.
Sclerosing cholangitis is associated with UC and can progress to cirrhosis and
hepatocellular failure.
Cholangiocarcinoma is rare association and its frequency is not influenced by colectomy.
The skin lesions include erythema nodosum and pyoderma gangrenosum.
The eyes can also be affected with uveitis and episcleritis.

Q-149 Which of the following feature is more common in Ulcerative colitis than Chrons
disease?
a. Perianal disease
b. Fistula formation
c. Stricture
d. Crypt abscess
Ans-149: (d) Crypt abscess

www.damsdelhi.com

Email: info@damsdelhi.com

DAMS CBT 2016 SET - 1

Ref: Read the text below


Sol:

Q-150 All of the following can cause paralytic ileus except?


a. Hypokalaemia
b. Hypocalcaemia
c. Salmonella typhi infection
d. Diffuse peritonitis
Ans-150: (b) Hypocalcaemia
Ref: Read the text below
Sol:
Common causes of paralytic ileus include
Post operative
Infection Intra peritoneal sepsis , Salmonella typhi infection
Reflex ileus - # of Ribs or spine,Retroperitoneal hemorrhage
Metabolic Uremia, Hypokalemia

Q-151 The sac contains only a portion of the circumference of the intestine is called
a. Richters Hernia
b. Littres hernia
c. Spigelian hernia
d. Lumber hernia
Ans-151: (a) Richters Hernia
Ref: Read the text below

www.damsdelhi.com

Email: info@damsdelhi.com

DAMS CBT 2016 SET - 1

Sol:
(1) Richters Hernia is a hernia in which the sac contains only a portion of the
circumference of the intestine (usually small intestine). It usually complicates femoral and
rarely obturator hernias.
(2) Littres hernia A Mechels diverticulum
(3) Spigelian hernia This is a variety of interparietal hernia occurring at the level of the
accurate line.
(4) Lumber hernia- Most primary lumber hernia occurs through the inferior lumber triangle
of Petit. Bounded below by the crest of the ilium, laterally by the external oblique and
medially by the latissmus dorsi.

Q-152 Millidgan-Morgan operation is indicated in


a. Rectal prolapsed
b. Haemorrhoides
c. Anal fissure
d. Fistula in Ano
Ans-152: (b) Haemorrhoides
Ref: Read the text below
Sol:
Indication of Haemorrhoidectomy
Third degree haemorrhoids
Second degree haemorrhoids which have not been cured by non operative treatments
Fibrosed haemorrhoids
Unteri external haemorrhoids when the external haemorrhoids is well defined
Haemorrhoidectomy can be performed using an open or closed technique-open
technique is most commonly used in the UK and is known as the Milligan-Morgan operation.
In the open technique the anal mucosa and skin are left open to heal by secondary
intension, and in the closed technique the wound is sutured.
Delormes operation used in rectal prolapsed

Q-153 Seat belt syndrome is


a. Fracture ilium with rupture of urethra
b. Sudden deceleration can result in a torn mesentery
c. Frature shaft of the femur with testicular swelling
d. Non-specific ; mesenteric adenitis
Ans-153: (b) Sudden deceleration can result in a torn mesentery
Ref: Read the text below
Sol:
SEAT- BELT SYNDROME
If a car accident occurs when a seat belt is worn, sudden deceleration can result in a
torn mesentery.
This possibility should be borne in mind, particularly as multiple injuries may distract
attention from this injury.

www.damsdelhi.com

Email: info@damsdelhi.com

DAMS CBT 2016 SET - 1

If there is any bruishing of the abdominal wall or even marks of clothing impressed into
the skin, laparotomy may be indicated.
Diagnostic peritoneal lavage may be helpful in this situation.

Q-154 Pneumatosis cystoids intestunalis is


a. Gas filled cysts in the subserosa or submucosa of the small intenstine
b. Gas filled cysts in the subserosa or submucosa of the duofenum
c. An external fistula communicating with the caecum following operation of gangrenous
appendicitis
d. Fibrotic thickening of the intestinal wall with narrow lumen.
Ans-154: (a) Gas filled cysts in the subserosa or submucosa of the small intenstine
Ref: Read the text below
Sol:
Gas filled cysts in the subserosa or submucosa of the small intenstine
Pneumatosis cystoids intestinalis- in which gas- filled cysts are found in the subserosa or
submucosa of the intestine or colon.
Majority of cases are associated with COPD or the immunocompromised state (eg. AIDS ,
post transplantation; in association with leukemia, lymphoma, vasculitis, or collagen
vascular disease, and in those patients taking chemotherapy or corticosteroids) other
associated conditions include inflammatory , obstructive or infectious conditions of the
intestine: iatrogenic conditions as such endoscopy and jejunstomy placement; ischemia
and extra-intestinal disease such as diabetes.

Q-155 Most common site of intra peritoneal abscess is:


a. Left superior intraperitoneal space
b. Left inferior intraperitoneal space
c. Right superior intraperitoneal space
d. Right inferior intraperitoneal space
Ans-155: (d) Right inferior intraperitoneal space
Ref: Read the text below
Sol:
Right inferior (posterior) intra peritoneal (right subhepatic")space: This lies transversely
beneath the right lobe of the liver in the Rutherford Morisons pouch
It is the deepest space of the four and the commonest site of a subphenic abscess which
usually arise from appendicitis, cholescystitis, a perforated duodenal ulcer or following
upper abdominal surgery.
The complicated arrangement of the peritoneum results in the formation of four
intraperitoneal and three extra peritoneal space in which pus may collect.

Q-156 Risk of permanent hypoparathyroidism after total thyroidectomy is about?


a. 1%
b. 2%

www.damsdelhi.com

Email: info@damsdelhi.com

DAMS CBT 2016 SET - 1

c. 5%
d. 10%
Ans-156: (c) 5%
Ref: Read the text below
Sol:
Risk of permanent hypoparathyroidism after total thyroidectomy is about 5%
In subtotal thyroidectomy it is 1%

Q-157 Positive predictive value of triple assessment of breast symptoms is about?


a. 85%
b. 95%
c. 65%
d. 99.9%
Ans-157: (d) 99.9%
Ref: Read the text below
Sol:

Q-158 All of the following statements about Phylloides tumour are true except?
a. Commonly seen after the age of 40.
b. Usually becomes large massive tumour.
c. Ulceration of overlying skin is common.
d. Can metastasis via lymphatics.
Ans-158: (d) Can metastasis via lymphatics.
Ref: Read the text below
Sol:
Phyllodes Tumour
Benign
Also known as serocystic disease of Brodie or cystosarcoma phyllodes
Usually occur in women over the age of 40 years
Present as a large, sometimes massive tumour
www.damsdelhi.com

Email: info@damsdelhi.com

DAMS CBT 2016 SET - 1

Ulceration of overlying skin occurs because of pressure necrosis.


Mobile on the chest wall
rarely cystic
Very rarely develop features of a sarcomatous tumour
May metastasise via the bloodstream.
Treatment for the benign type is enucleation or wide local excision.
Massive tumours, recurrent tumours and those of the malignant type will require
mastectomy

Q-159 Which of the following is ANDI?


a. Galactocoele
b. Duct ectasia
c. Puerperal abscess
d. Cyclical mastalgia
Ans-159: (d) Cyclical mastalgia
Ref: Read the text below
Sol:
ANDI (aberations of normal differentiation and involution) includes
Cyclical nodularity and mastalgia
Cysts
Fibroadenoma

Q-160 After Pateys mastectomy the resected specimen does not contain?
a. Whole breast
b. Skin over the breast along with nipple areola complex
c. All lymphnodes in axilla
d. Pectoralis minor
Ans-160: (d) Pectoralis minor
Ref: Read the text below
Sol:
Patey mastectomy
The breast and associated structures are dissected en bloc and the excised mass is
composed of:
o the whole breast;
o a large portion of skin, the centre of which overlies the tumour but which always includes
the nipple;
o all of the fat, fascia and lymph nodes of the axilla.
The pectoralis minor muscle is either divided or retracted
The axillary vein and nerves to the serratus anterior and latissimus dorsi (the
thoracodorsal trunk) should be preserved.
The intercostal brachial nerves are usually divided
Early mobilisation of the arm is encouraged

www.damsdelhi.com

Email: info@damsdelhi.com

DAMS CBT 2016 SET - 1

Q-161 In CEAP classification lipodermatosclerosis is classified as?


a. C3
b. C4a
c. C4b
d. C5
Ans-161: (c) C4b
Ref: Read the text below
Sol:
The CEAP (clinical etiology anatomy pathophysiology) classifi cation
Clinical classifi cation
C0: no signs of venous disease
C1: telangectasia or reticular veins
C2: varicose veins
C3: oedema
C4a: pigmentation or eczema
C4b: lipodermatosclerosis or atrophie blanche
C5: healed venous ulcer
C6: active venous ulce
Etiologic classifi cation
Ec: congenital
Ep: primary
Es: secondary (post-thrombotic)
En: no venous cause identifi ed
Anatomical classifi cation
As: superfi cial veins
Ap: perforator veins
Ad: deep veins
An: no venous location identifi ed
Pathophysiological classifi cation
Pr: refl ux
Po: obstruction
Pr,o: refl ux and obstruction
Pn: no venous pathophysiology identifi able

Q-162 Base of the skull fracture presents with involvement of the petrous temporal bone,
which of the following important sign:
a. Subconjunctive haematoma
b. CSF rhinorrhoea
c. Raccon eyes
d. Battle sign
Ans-162: (d) Battle sign
Ref: Read the text below
Sol:
www.damsdelhi.com

Email: info@damsdelhi.com

DAMS CBT 2016 SET - 1

Base of Skull Fractures


Anterior fossa fracture- May open into the frontal or ethmoid air sinuses or run across
the cribriform plate.
Presents with sub conjunctival hematoma,anosmia, epistaxis, nasal tip anaesthesia, CSFrhinorrhea and occasionally carotico-cavernous fistua.
Periobital haematoma or raccoon eye indicate subgaleal haemorrhage.
Middle fossa fracture involving petrous temporal bone presents with CSF otorrhea,
haemotympanum, occicular disruption, Battle sign or VII and VIII cranial nerve palsies.
Battle sign bruising behind the ear appearing 36 hours after a head injury with a petrous
temporal base of skull fracture.
Q-163 Wolfe grafts is :
a. Full thickness skin grafts
b. Partial thickness grafts
c. Split skin grafts
d. Pedicled flap
Ans-163: (a) Full thickness skin grafts
Ref: Read the text below
Sol:
Split-skin grafts
(THIERSCH GRAFTS)
Donor site factors
Large area available
Donor site heals spontaneously
Donor site reusable
Recipient factors
Poor color match
Easily abraded
Inferior cosmetic resistant
More reliable take
Shiny texture, inelastic

FULL thickness grafts


(WOLFF-GRAFTS)
- Small area
- Donor site must be closed
- Donor site scars

Good colour match


Abrasion resistant
Good cosmetic resistant
Less reliable take
Normal texture elastic

Q-164 Crumbled egg appearance in liver seen in :


a. Hepatic adenoma
b. Chronic amobic liver abscess
c. Hydatid liver of disease
d. Haemagioma
Ans-164: (c) Hydatid liver of disease
Ref: Read the text below
Sol:
Cysts due to echinococcus (hydatid) disease may be single or multiple , a few show,
calcified walls.

www.damsdelhi.com

Email: info@damsdelhi.com

DAMS CBT 2016 SET - 1

Hydatid liver cyst- Active hydatid disease usually produces a non-calcified liver cyst and
within the cysts-floating layers of the germinal membrance can be seen.

Q-165 The best investigation to assess complex incisional hernia is?


a. Xray
b. USG
c. CT Scan
d. MRI Scan
Ans-165: (c) CT Scan
Ref: Read the text below
Sol:
Computed tomography scanning is helpful in complex incisional hernia, determining the
number and size of muscle defects, identifying the content, giving some indication of
presence of adhesions and excluding other intra-abdominal pathology such as ascites,
occult malignancy, portal hypertension, etc.
Q-166 Chilotic line is?
a. Line on hip bone for sex determination
b. Line on femur for sex determination
c. Line on hip bone for race determination
d. Line in femur bone for race determination
Ans-166: (a) Line on hip bone for sex determination
Ref.:Read the text below
Sol :
Chilotic line is a line on hip bone for sex determination.
A line is extended back from the iliopectineal eminence to the nearest point on the
anterior auricular margin and thence to the iliac crest.
The auricular point divides this chilotic line into anterior (pelvic) and posterior (sacral)
segments, each expressed as a percentage of the other.
Chilotic indices display reciprocal values in the sexes : the pelvic part of the chilotic line
is predominant in females (63 mm in males, 67.3 mm in females), and the sacral part in
males (65.5 in males, 55.6 in females).

Q-167 Takayama reagent is used in detection of?


a. Semen
b. Blood
c. Gun powder
d. Old tattoos
Ans-167: (b) Blood
Ref.:Read the text below
Sol :
TESTS USED FOR DETECTION OF BLOOD
Test
Comments

www.damsdelhi.com

Email: info@damsdelhi.com

DAMS CBT 2016 SET - 1

Takayamas Haemochromagen crystal


test
Kastle-Mayer-phenolphthalein test
Teichmanns Hemin crystal test
Benzidine test (gicial acetic acid +
H2O2)

Spectroscopic test
Thin layer chromatography
Electrophoresis

Salmon pink colour haemochromagen crystals are


obtained
Less sensitive but more specific for blood
Brown rhombic hemin crystals are seen
Greenish blue colour obtained
Positive even with blood subjected to heat or cold
Negative reaction rules out presence of blood
Best preliminary test (detects blood in 1:3 lakh
dilution)
Most specific

Q-168 The word subpoena is synonymous with :


a. Res ipsa loquitur
b. Res judicata
c. Summons
d. Solemn affirmation
Ans 168: (c) Summons
Ref:Read the text below
Sol:
A subpoena is a writ issued by a government agency that has authority to
compel testimony by a witness or production of evidence, the agency most often acourt,
under a penalty for failure.

Q-169 Guainacum test gives which color in the presence of blood


a. Pink
b. Deep blue
c. Bright green
d. Yellowish brown
Ans 169: (b) Deep blue
Ref:Read the text below
Sol:
Six color tests have been traditionally used by forensic scientists to detect blood stains. All
these tests use the peroxidase like activity of heme to knock a nascent oxygen atom off
hydrogen peroxide. This nascent oxygen atom brings about a color change in various
reagents. These color changes have often been asked in examinations. Various screening
tests for blood and the colors they give
S.No.
Reagent
Color change
1.
Benzidime
Intense Blue
2.
Ortho-tolidine
blue green
3.
Tetramenthylbenzidine
green or blue green
(TMB)

www.damsdelhi.com

Email: info@damsdelhi.com

DAMS CBT 2016 SET - 1

4.
5.
6.

Phenolphthalein
(Kastle Meyer test)
Leucomalachite green
Guaicum

Pink
Bright green
Deep blue

Q-170 Skull of a male differs from that of a female by all of the following except.
a. Capacity is greater than 1500 cc in males
b. Muscular markings over occiput are less marked in males.
c. Orbits are square in males.
d. Frontal eminence is smaller in males
Ans 170: (b) Muscular markings over occiput are less marked in males.
Ref:Read the text below
Sol:
Skull difference between Male and Female
The male cranial mass is more blocky and massive compared to the females which
more rounder and tapers at the top.
Capacity is greater than 1500 cc in males
Muscular markings over occiput are more marked in males.
Orbits are square in males.
Frontal eminence is smaller in males
The females Supraorbital margin is sharper while the males is rather round and dull.
The Zygomatic bone is more pronounced on the male skull.
The Mandible of a female is more rounded while the male's is squared.
Males have a deeper cranial mass
The supercilary arch is large and pronounced in the male.

Q-171 Mechanical obstruction of the flow of air from the environment into the mouth
and/or nostril is ?
a. Smothering.
b. Guillotine
c. Garroting
d. Burking
Ans 171: (a)Smothering.
Ref:Read the text below
Sol:
Smothering is the mechanical obstruction of the flow of air from the environment into
the mouth and/or nostrils, for instance, by covering the mouth and nose with a hand, pillow,
or a plastic bag.
In homicidal cases, the term burking is often ascribed to a killing method that involves
simultaneous smothering and compression of the torso.
They killed the usually-intoxicated victims by sitting on their chests and suffocating
them by putting a hand over their nose and mouth, while using the other hand to push
the victim's jaw up.

www.damsdelhi.com

Email: info@damsdelhi.com

DAMS CBT 2016 SET - 1

Q-172 A dumb witness :


a. Is not allowed to testify in a court of law
b. Can testify by signs in an open court.
c. By producing a previously written statement.
d. By writing in an open court.
Ans 172: (b) Can testify by signs in an open court.
Ref:Read the text below
Sol:
DUMB WITNESSES ( S. 119 IEA )
A witness who is unable to speak may give his evidence in any other manner in which
he can make it intelligible, as by writing or by signs; but such writing must be written and
the signs made in open Court.
Evidence so given shall be deemed to be oral evidence.

Q-173 All of the following are related to medical negligence except :


a. Section 304A I.P.C.
b. Section 37 I.P.C.
c. Section 34I I.P.C.
d. Section 312 I.P.C.
Ans 173: (b) Section 37 I.P.C.
Ref:Read the text below
Sol:
SECTION 37 I.P.C.
Co-operation by doing one of several acts constituting an offence
When an offence is committed by means of several acts, whoever intentionally cooperates in the commission of that offence by doing any one of those acts, either singly or
jointly with any other person commits that offence.

Q-174 Whoever causes the death of any person by doing any rash or negligent act not
amounting to culpable homicide, shall be punished with imprisonment of either
description for a term which may extend to two years, or with fine, or with both is:
a. Section 304A I.P.C.
b. Section 37 I.P.C.
c. Section 34I I.P.C.
d. Section 312 I.P.C.
Ans 174: (a) Section 304A I.P.C.
Ref:Read the text below
Sol:
Whoever causes the death of any person by doing any rash or negligent act not
amounting to culpable homicide, shall be punished with imprisonment of either description
for a term which may extend to two years, or with fine, or with both

www.damsdelhi.com

Email: info@damsdelhi.com

DAMS CBT 2016 SET - 1

Q-175 Testing for the presence of air in the middle ear is


a. Foderes test
b. Ploucquets test
c. Breslaus second life test
d. Wredens test
Ans 175: (d) Wredens test
Ref:Read the text below
Sol:
During embryonic life, the middle ear contains gelationous tissue.
During afforts of breathing, some air enters the middle ear through the Eustachian
tube.
Thus if the air is found in the middle ear, it is quite reasonable to suppose that the
child had been born alive.

Q-176 Maximum analgesic action is seen with:


a.
Catecholamine
b.
Propofol
c.
Ketamine
d.
Thiopentone
Ans 176: (c) Ketamine
Ref Read the text below
Sol:

Ketamine produces profound analgesia. It produces a state where one feels


dissociated from his own body.

It does not produce unconsciousness. Propofol and thiopentone are the induction
agaents causing unconsciousness quickly.

They are poor analgesics. Catecholamines e.g. Adrenaline, Noradrenaline do not


produce analgesia.

Q-177 Anesthetic agent of choice in renal failure is:


a.
Methoxyflurane
b.
Isoflurane
c.
Enflurane
d.
None of the above
Ans 177: (b) Isoflurane
Ref Read the text below
Sol:

Methoxyflurane and enflurane are metabolized in the body producing inorganic


fluoride which is quite toxic to the renal tubules and can cause high-output renal
failure;hence contra-indicated in renal failure.

www.damsdelhi.com

Email: info@damsdelhi.com

DAMS CBT 2016 SET - 1

Because of this problem, they are practically out of use nowadays.

Since isoflurane is sufficiently lipid insoluble; it is exhaled back through respiration


before any significant metabolism occurs.
Q-178 Which one of the following statements regarding desflurane is correct?
a.
It causes severe myocardial depression
b.
It is a structural analogue of isoflurane
c.
It has a very high blood and tissue-gas partition coefficients
d.
It is metabolically unstable
Ans-178: (b) It is a structural analogue of isoflurane
Ref:Morgans Anesthesia -144
Sol :

Desfluranes structure is very similar to Isoflurane;

It is highly volatile; Very low solubility in blood & tissues i.e, very low B/G partition
coefficient, thus induction & recovery are very fast.

It causes minimal cardiac depression & undergoes minimal metabolism

Q-179 All of the following statements are incorrect regarding treatment of prolonged
suxamethonium apnea due to plasma cholinesterase deficiency (after a single dose of
suxamethonium) except
a.
Reversal with incremental doses of neostigmine
b.
Continue anesthesia & mechanical ventilation till recovery
c.
Transfusion of fresh frozen plasma
d.
Plasmapheresis
Ans-179: (a) Reversal with incremental doses of neostigmine
Ref:Lees Anesthesia -.223]
Sol :

The best management in this case is to provide mechanical ventilation, maintain


anesthesia and continue monitoring till muscle function returns to normal.

Transfusion of fresh frozen plasma is beneficial (It will provide pseudocholinesterase)

Q-180 Differential sensory blockade is seen with


a.Lignocaine
b.
Ropivacaine
c. Prilocaine
d.
Tetracaine
Ans-180: (b) Ropivacaine
Ref:Read the text below
Sol :

Differential blockade occur with bupivacaine and ropivacaine.

At lower concentration only sensory blockade while at higher concentration both


motor and sensory

www.damsdelhi.com

Email: info@damsdelhi.com

DAMS CBT 2016 SET - 1

Q-181 Most common artery used for invasive monitoring is


a.Ulnar artery
b.
Dorsalis pedis artery
c. Femoral artery
d.
Radial artery
Ans-181: (d) Radial artery
Ref:Read the text below
Sol :

Radial artery is used for invasive monitoring.

Allens test is used to confirm collateral circulation

Q-182 A rapid fall in ETCO2 is a indicator for


a.Malignant hyperthermia
b.
Accidental extubation
c. Venous air embolism
d.
Disconnection in circuit
Ans-182: (c) Venous air embolism
Ref:Read the text below
Sol :

Rapid fall in ETCO2 is indicative of venous air embolism which occurs most commonly
in posterior fossa surgery in sitting position.

Q-183 Choice of induction agent for LMA insertion is :a.Thiopentone


b.
Propofol
c. Halothane
d.
Althesin
Ans-183: (b) Propofol
Ref:Read the text below
Sol :

Propofol Causes Maximum Depression Of Upper Airway Reflexes So Choice For Lma
Insertion

Q-184 Propofol infusion syndrome consist of all except


a.Nephropathy
b.
Acute cardiomyopathy
c. Metabolic acidosis
d.
Skeletal myopathy
Ans-184: (a) Nephropathy
Ref:Read the text below
Sol :

www.damsdelhi.com

Email: info@damsdelhi.com

DAMS CBT 2016 SET - 1

Propofol infusion syndrome consists of metabolic acidosis, skeletal myopathy and


acute cardiomyopathy.

It occurs in children on prolonged infusion due to failure of metabolism of free fatty


acids.

Q-185 Ideal muscle relaxant for use of continuous infusion is


a.Rocuronium
b.
Mivacurium
c. Pancuronium
d.
Vecuronium
Ans-185: (b) Mivacurium
Ref:Read the text below
Sol :

The ideal muscle relaxant for continous infusion is mivacurium as it has got ultra short
duration of action.

It has a slow onset.


Q-186 All of the following are true except :
a. Osteomyelitis of the bone can spread through human and animal bites
b. In a suspected osteomyelitis , first step is to withdraw the blood and fluid samples rather
than to start broad spectrum intravenous antibiotics
c. The most common complication of acute osteomyelitis is chronic osteomyelitis
d. Development of malignancy is a very common complication of acute osteomyelitis
Ans 186: (d) Development of malignancy is a very common complication of acute
osteomyelitis
Ref: Read the text below
Sol:

Go through the most common causes of osteomyelitis in different clinical scenarios.

Sinus tract malignancy is a common complication of chronic osteo myelitis and not
acute osteomyelitis

Q-187 All of the following are the components of terrible triad of Hotchkiss except :
a. Fracture radial head
b. Posterior dislocation of elbow
c. Fracture of the coronoid process
d. Fracture of the olecranon process
Ans 187: (d) Fracture of the olecranon process
Ref: Read the text below
Sol:

Fracture of olecranon process is not a part of terrible triad of Hotchkiss.


Terrible triad of Hotchkiss except :
a. Fracture radial head

www.damsdelhi.com

Email: info@damsdelhi.com

DAMS CBT 2016 SET - 1

b. Posterior dislocation of elbow


c. Fracture of the coronoid process

Q-188 This X ray given along represents which of the following terms

a. Arthrodesis
b. Arthroplasty
c. Arthrotomy
d. Ankylosis
Ans 188: (a) Arthrodesis
Ref: Read the text below
Sol:

Ankylosis : pathological fusion of a joint

Arthrodesis: surgical fusion of a joint

Q-189 A 55-year-old right-handed woman has left elbow pain laterally. X-ray findings are
negative. There is tenderness and slight swelling over the lateral epicondyle of the
humerus. Anatomically, this condition can be explained by which of the following?
a.
Sprain of the lateral collateral elbow ligament
b.
Rupture of the triceps muscle
c.
Tendinitis of the wrist extensors
d.
Synovitis of the left elbow joint
Ans 189: (c) Tendinitis of the wrist extensors
Reference: Read the text below
Sol:

The act of wringing rags results in repeated and forceful wrist dorsiflexion, causing
increased pressure on the wrist extensor muscles, which have their tendinous origins from
the lateral humeral epicondyle.

This results in an inflammatory condition at the bone tendon junction, lateral


epicondylitis, or tennis elbow.

Although this condition is common in tennis players, it occurs more frequently in


the general population.

www.damsdelhi.com

Email: info@damsdelhi.com

DAMS CBT 2016 SET - 1

Q-190 An86-year-old woman experiences left hip pain after a fall at home. She cannot
ambulate,her hip area is swollen and painful, and her left lower extremity is shortened and
externallyrotated. Before the fall, she was ambulatory and had no complaint of hip,
pelvic, or kneepain. In addition to the fracture of the proximal portion of the left femur,
the x-ray would show which of the following?
a.
Arthritis of the left hip
b.
Calcific bursitis of the left hip
c.
Osteoporosis
d.
Fracture of the pelvis
Ans 190: (c) Osteoporosis
Reference: Read the text below
Sol:

Postmenopausal osteoporosis is the common denominator in all fractures involving


elderly women.

In this particular fracture, it is the twisting effect on an osteoporotic femur that


causes the fracture rather than the impact of the fall itself.

Q-191 Which of the following is osteoinductive :


a. Cancellous allograft
b. Cancellous autograft
c. Hydroxyapatite
d. Collagen based matrix
Ans 191: (b) Cancellous autograft
Ref: Read the text below
Sol:

An ideal bone-graft substitute must provide scaffolding for osteoconduction as well


as progenitor cells and growth factors for osteoinduction.

Furthermore, the bone graft must be able to integrate with the host.

Autogenous bone graft contains osteoblasts, endosteal osteoprogenitor cells


capable of synthesizing new bone, and a structural matrix that acts as a scaffold, making it
the gold standard for bone grafting.

BMP-2 is a commonly utilized adjunct for grafting, and is inherently osteoinductive.

The referenced article by Buckwalter et al is a review on the biology of bone


grafting which nicely defines the various osteoinductive and osteoconductive properties of
the various bone graft options.

Q-192 Most important technical consideration at the time of doing a below Knee
Amputation is :
a. Stump should be kept too long
b. Stump should be kept too short
c. Anterior flap should be kept longer than the posterior flap
d. Posterior flap should be kept longer than the anterior flap
Ans 192: (d) Posterior flap should be kept longer than the anterior flap

www.damsdelhi.com

Email: info@damsdelhi.com

DAMS CBT 2016 SET - 1

Ref: Read the text below


Sol:
Techniques to follow amputation :
- Flap is longer on the posterior side than anterior side
- nerve and vessels should be doubly ligated
- suture line should be obtained anterior to the midline in coronal plane
- level of amputation must be planned earlier

Q-193 Most common nerve injured in Supracondylar fracture of Humerus :


a. Anterior Interosseous Nerve
b. Median Nerve
c. Radial Nerve
d. Ulnar Nerve
Ans 193: (a) Anterior Interosseous Nerve
Ref: Read the text below
Sol:
Anterior Interosseous nerve a branch of median nerve is most commonly injured nerve
in supracondylar fracture humerus while tardy ulnar nerve palsy is most commony seen in
fracture lateral condyle humerus Circumflex branch of axillary nerve is most commonly
injured in fracture proximal humerus.

Q-194 Which is the most common carpal bone to get dislocated :


a. Scaphoid
b. Lunate
c. Pisiform
d. Hamate
Ans 194: (b) Lunate
Ref: Read the text below
Sol:
Scaphoid is the most commonly injured bone among carpal bones and lunate is the
most commonly dislocated.
Scaphoid has so many exceptions and one important one is Retrograde blood flow
which leads to avascular non union

Q-195 Which is the first reflex to recover after a period of Spinal Shock :
a. Abdominal reflex
b. Ankle jerk
c. Cremasteric reflex
d. Bulbocavernous reflex
Ans 195: (d) Bulbocavernous reflex
Ref: Read the text below
Sol:

www.damsdelhi.com

Email: info@damsdelhi.com

DAMS CBT 2016 SET - 1

Bulbocavernous reflex is the first reflex to recover after spinal shock though not
very clearly appreciable.
Q-196 The vector shown below transmits which of the following diseases?

a. Malaria
b. Filaria
c. Yellow fever
d. Kala-azar
Ans-196: (d) Kala-azar
Ref.:Read the text below
Sol :
Sandfly (Phlebotomus) is the vector of Kala-azar
The average life of a sandfly is about 2 weeks.
Diseases transmitted by Sandfly
SPECIES
Phlebotomusargentipes
Phlebotomuspapatasii
Phlebotomussergenti

DISEASES TRANSMITTED
Kala-azar
Sandfly fever
Oriental sore
Oriental sore

Q-197 A patient presents with lower gastrointestinal bleed. Sigmoidoscopy shows ulcers
in the sigmoid. Biopsy from this area shows flask-shaped ulcers. Which of the following is
the most appropriate treatment?
a. Intravenous ceftriaxone
b. Intravenous metronidazole
c. Intravenous steroids and sulphasalazine
d. Hydrocortisone enemas
Ans-197: (b) Intravenous metronidazole
Ref.:Read the text below
www.damsdelhi.com

Email: info@damsdelhi.com

DAMS CBT 2016 SET - 1

Sol :
It is case of intestinal amoebiasis in the form of amoebic dysentery
Drug Therapy for Amebiasis
Asymptomatic carrier
Acute Colitis
Amebic liver abscess
(Luminal agents)
Lodoquinol
Metronidazole plus
Metronidazole or
Paromomycin
Luminal agent
Tinidazole or tinidazole
or ornidazole plus Luminal
agent

Q-198 The following is characteristic feature of staphylococcus food poisoning except:


a. Optimum temperature for toxin production is 37C
b. Intradietic toxin are responsible for intestinal symptoms
c. Toxin can be destroyed by boiling for 30 minutes
d. Incubation period is 1-6 hours.
Ans-198: (c) Toxin can be destroyed by boiling for 30 minutes
Ref.:Read the text below
Sol :
Staphylococcal food poisoning :
Staphylococcal food poisoning is due to heat stable preformed toxin mostly after
consuming milk products.
Toxin is produced optimally at 35C to 37C
IP- 1.-6 hours.
Mechanism Acts by stimulating vagus nerve and vomiting center of brain

Q-199 Which of the following infestations leads to malabsorption?


a. Giardia lamblia
b. Ascaris lumbricoides
c. Necator Americana
d. Ancylostoma duodenale
Ans-199: (a) Giardia lamblia
Ref.:Read the text below
Sol :
Giardia lamblia

Symptoms of infection include (in order of frequency) diarrhea, malaise, excessive gas
(often flatulence or a foul or sulphuric-tasting belch, which has been known to be so
nauseating in taste that it can cause the infected person to vomit), steatorrhoea (pale, foul
smelling, greasy stools), epigastric pain, bloating, nausea, diminished interest in food,
possible (but rare) vomiting which is often violent, and weight loss
People with recurring Giardia infections, particularly those with a lack of the
immunoglobulin A antibody, may develop chronic disease.
Lactase deficiency may develop in an infection with Giardia, but this usually does not
persist for more than a few weeks, and a full recovery is the norm.

www.damsdelhi.com

Email: info@damsdelhi.com

DAMS CBT 2016 SET - 1

Some studies have shown giardiasis should be considered as a cause of vitamin B12
deficiency as result of the problems caused within the intestinal absorption system
Ascaris lumbricoides in small intestine usually cause no symptoms. In children it may
cause pain and intestinal obstruction sometimes complicated by perforation,
intussusceptions or volvulus.
Migration to aberrant site can cause biliary colic, cholecystitis, cholangitis, pancreatitis
or rarely intrahepatic abscess.
Intestinal phase of A. duodenale cause epigastric pain, inflammatory diarrhea and iron
deficiency anemia.

Q-200 Disinfectant used as a standard of measurement of killing power.


a. Alcohol
b. Phenol
c. Cresol
d. Formaldehyde
Ans-200: (b) Phenol
Ref.: Read the text below
Sol :
Chemical disinfectants vary greatly in their ability to kill microorganisms.
A quantitative measure of this variation is expressed as the phenol coefficient, which is
the ratio of the concentration of phenol to the concentration of the agent required to cause
the same amount of killing under the standard conditions of the test.

Q-201 Which of the following is most resistant to antiseptics?


a. Spore
b. Prion
c. Cyst
d. Fungus
Ans-201: (b) Prion
Ref.:Read the text below
Sol :
Resistance of organism to antiseptics in decreasing order is as follows :
Prions
Coccidia
Spores
Mycobacteria
Cysts
Small non-enveloped virus
Trophozoites
Gram-negative bacteria
Fungi
Large non-enveloped virus

www.damsdelhi.com

Email: info@damsdelhi.com

DAMS CBT 2016 SET - 1

Gram-positive bacteria
Lipid enveloped/medium size virus (HIV, HBV)

Q-202 Which of the following statement is true:


a. Solid media are enrichment media
b. Nutrient broth is basal media
c. Agar adds nutrient to media
d. Chocolate agar is selective media.
Ans-202: (b) Nutrient broth is basal media
Ref.:Read the text below
Sol :
Media
Simple media
Complex media
Synthetic or defined
(Basal media)
media
Nutrient broth
Added ingredient
Prepared from
chemicals with
defined composition
e.g., simple peptone
water medium

Special media
Enriched media
Enrichment media
Selective media
Indicator media
Sugar media
Transport media

Q-203 In all of the following diseases chronic carriers are found except :
a. Measles
b. Typhoid
c. Hepatitis B
d. Gonorrhea
Ans-203: (a) Measles
Ref.:Read the text below
Sol :
In measles and whooping cough only cases are found with no carriers.
Chronic carriers seen in :
Typhoid
Dysentery
Hepatitis B
Malaria
Cerebrospinal meningitis
Gonorrhea

Q-204 A woman with infertility receives an ovary transplant from her sister who is an
identical twin. What type of graft is it?
a. Xenograft
b. Autograft

www.damsdelhi.com

Email: info@damsdelhi.com

DAMS CBT 2016 SET - 1

c. Allograft
d. Isograft
Ans-204: (d) Isograft
Ref.:Read the text below
Sol :
Terminology of grafts
Donor
Self
Different individual, genetically identical
with recipient. Identical twin or member
of same inbred strain.
Genetically unrelated member of same
species Different species

Term
Autograft
Isograft

Synonyms
Autogenous or autogenic graft
Autogenous or autogenic graft

Allograft
Xenograft

Isologous or syngeneic graft or


syngraft Xenogeneic. Formerly
called heterograft

Q-205 Rabies virus persists in the saliva for how many days:
a. 3-6
b. 7-10
c. 1-3
d. Upto 6 years
Ans-205: (a) 3-6
Ref.:Read the text below
Sol :
SOURCE OF RABIES INFECTION
The source of infection to man is the saliva of rabid animals.
In dogs and cats, the virus may be present in the saliva for 3-4 days (occasionally 5-6
days) before the onset of clinical symptoms and during the course of illness till death.
However, it may be stated that not all rabid animals have virus in saliva and even if
present, the quantity is variable.
The variability of the virus in saliva explains the fact why only about 50% of bites by
proven rabid animals will result in rabies.

Q-206 Tubercle bacilli was discovered by:


a. Robert Koch
b. Hansen
c. Edward Jenner
d. Virchow
Ans-206: (a) Robert Koch
Ref.:Read the text below
Sol :
On March 24, 1882, Robert Koch announced to the Berlin Physiological Society that he
had discovered the cause of tuberculosis.

www.damsdelhi.com

Email: info@damsdelhi.com

DAMS CBT 2016 SET - 1

Three weeks later, on April 10, he published an article entitled "The Etiology of
Tuberculosis"
In 1884, in a second paper with the same title, he first expounded "Koch's postulates,"
which have since become basic to studies of all infectious diseases.
He had observed the bacillus in association with all cases of the disease, had grown the
organism outside the body of the host, and had reproduced the disease in a susceptible
host inoculated with a pure culture of the isolated organism.

Q-207 Which one of the following Gram positive organism is most common cause of UTI
among sexually active women:
a. Staphylococcus epidermidis
b. Staphylococcus aureus
c. Staphylococcus saprophyticus
d. Enterococcus
Ans-207: (c) Staphylococcus saprophyticus
Ref.:Read the text below
Sol :
S. saprophyticus cause UTI in young women due to its enhanced capacity to adhere to
uroepithelial cells.
A 160 KDa hemagglutinin adhesions may contribute to this affinity.
S. Saprophyticus is novobiocin resistant.

Q-208 In autoclaving, which of the following spore used to confirm effectiveness:


a. Bacillus anthrax
b. Bacillus stearothermophilus
c. Clostridia spores
d. Bacillus cerius
Ans-208: (b) Bacillus stearothermophilus
Ref.:Read the text below
Sol :
STERILIZATION CONTROL :
For determining the efficacy of moist-heat sterilization, spores of Bacillus
stearothermophilus are used as the test organism.
The is a thermophilic organism with an optimum growth temperature of 55-60C and
its spores require an exposure of 12 minutes at 121C to be killed.
Paper strips impregnated with 106 spores are dried at room temperature and placed in
paper envelopes.
These envelopes are inserted in different parts of a load and after being sterilized; the
strips are inoculated into a suitable recovering medium and incubated for sterility test at
55C for five days.
Chemical indicators, autoclave tapes and thermocouples are also used instead.

www.damsdelhi.com

Email: info@damsdelhi.com

DAMS CBT 2016 SET - 1

Q-209 Which of the following cannot bereliably used for hand washing :
a. Chlorh exidine
b. Isopropyl alcohol
c. Lysol
d. Cresol
Ans-209: (c) Lysol
Ref.:Read the text below
Sol :
Skin disinfectants are :
Chlorhexidine (Savlon)
Alcohols (as spirit) Isopropyl alcohol (preferred), ethyl alcohol
Iodine
Cresol
Chloroxylenol
Hexachlorophene
Tincture iodine

Q-210 Which component of streptococcal pyogenes has cross reactivity with synovium of
human?
a. Capsular hyaluronic acid
b. Cell proteins
c. Group A carbohydrate antigens
d. Peptidoglycan
Ans-210: (a) Capsular hyaluronic acid
Ref.:Read the text below
Sol :
Gross Reactivity of streptococcal antigen
Bacterial antigen
Cross reacting human component
Capsular hyaluronic acid
Human synovial fluid
Group A carbohydrate antigen
Cardiac valves
Cytoplasmic membrane antigen
Vascular intima
Cell wall protein
Myocardium
Peptidoglycan
Skin antigen
Membrane antigens
Sarcolemma of smooth and cardiac
muscle, dermal fibroblasts
And neurons of caudate nucleus

Q-211 An HIV positive female has an indurated ulcer over cornmeal agar at 20 degrees,
microscopy showing hyphae and growth inhuman serum at 37 degrees show budding
yeasts. The probable cause is:
a. Candida albicans
b. Histoplasmosis
c. Blastomycosis

www.damsdelhi.com

Email: info@damsdelhi.com

DAMS CBT 2016 SET - 1

d. Coccidiodomycosis
Ans-211: (a) Candida albicans
Ref.:Read the text below
Sol :
This is a case of oral thrush secondary to candidiasis
C. albicans is a dimorphic fungi which occur both as yeast and moulds (with hyphae)
In HIV oral thrush occurs when CD4 < 50/ul.

Q-212 Regarding HIV which of the following is not true?


a. It is a DNA retrovirus
b. Contains Reverse transcriptase
c. May infect host CD 4 cells other than T lymphoctyes
d. Causes a reduction in host CD 4 cells at late stage of disease
Ans-212: (a) It is a DNA retrovirus
Ref.:Read the text below
Sol :
HIV is RNA retrovirus (not DNA virus)
It primarily infect CD 4 + T cells but can infect other cells also which bear CD4 receptor
on their surface
These include circulating dendritic cells; epidermal langerhans cells; monocytes.
Q-213 Which toxin of streptococcus causes hemolysis
a. Streptolysin O
b. Streptolysin S
c. Streptokinase
d. Streptodornase
Ans-213: (b) Streptolysin S
Ref.:Read the text below
Sol :
Streptococci Produce two hemolysin
Streptolysin O
Streptolysin S (Serum soluble)
Oxygen labile
Oxygen stable
Activity only on pour plate not on
Responsible for hemolysis seen around
surface
streptococcal colonies on surface
Antigenic protein
Non antigenic protein elaborated in
presence of serum

Q-214 A patient admitted to an ICU is on central venous line for the last one week. He is
on ceftazidime and amikacin. After 7 days of antibiotics he develops a spike of fever and his
blood cultyure is positive for gram positive cocci in chains which are catalase-negative.
Following this vancomycin was started but the culture remained positive for same organism
even after 2 weeks of therapy. The most likely organism causing infection is :
a. Staphylococci aureus

www.damsdelhi.com

Email: info@damsdelhi.com

DAMS CBT 2016 SET - 1

b. Viridans streptococci
c. Enterococcus faecalis
d. Coagulase negative staphylococcus
Ans-214: (c) Enterococcus faecalis
Ref.:Read the text below
Sol :
Enterococci are catalase negative and grow in chains and above all resistant to
cephalosporins.
Enterococci is a frequent cause of nosocomial bacteremias and many of these
enterococci are resistant to vancomycin.
Enterococcal bacteremias is characteristically seen in ICU in patient taking
cephalosporin as antibiotic.

Q-215 Which infection is not common in HIV patients?


a. Cryptosporidiosis
b. Atypical mycobacterial infection
c. Aspergillosis
d. Candidiasis
Ans-215: (c) Aspergillosis
Ref.:Read the text below
Sol :
All of the following organism can cause opportunistic infection in AIDS patient.
But invasive aspergillosis is generally not seen in patient with AIDS in the absence of
neutropenla or administration of glucocorticoids.
Aspergillosis also given in list of opportunistic infection but is not much common,
hence is best option.
Q-216 All of the following are causes of night blindness except :
a. Oguchi's disease
b. Gyrate atrophy
c. Choroideremia
d. Devics disease
Ans-216: (d) Devics disease
Ref: Read the text below
Sol:
Neuromyelitis optica (NMO), also known as Devic's disease or Devic's syndrome, is an
heterogeneous condition consisting of recurrent and simultaneous inflammation and
demyelination of the optic nerve (optic neuritis) and the spinal cord (myelitis).
The main symptoms of Devic's disease are loss of vision and spinal cord function.
Optic neuritis may manifest as visual impairment with decreased visual acuity, although
visual field defects, or loss of color vision may occur in isolation or prior to formal loss of
acuity.

www.damsdelhi.com

Email: info@damsdelhi.com

DAMS CBT 2016 SET - 1

Q-217 The diagnosis is :

a. Anterior lenticonus
b. Posterior lenticonus
c. Anterior lentiglobus
d. Posterior lentiglobus
Ans-217: (a) Anterior lenticonus
Ref: Read the text below
Sol:
This is view through slit lamp in retro illumination showing oil globule reflex , typical
of anterior lenticonus .

Q-218 Most common intracranial tumor encroaching the orbit is :


a. Astrocytoma
b. Glioblastoma multiforme
c. Sphenoid wing meningioma
d. Medulloblastoma
Ans-218: (c) Sphenoid wing meningioma
Ref: Read the text below
Sol:
Meningioma is a benign brain tumour arising from the arachnoid. It is more common in
females after the age of 50yrs.
20% of meningiomas are sphenoid wing meningiomas.

www.damsdelhi.com

Email: info@damsdelhi.com

DAMS CBT 2016 SET - 1

Q-219 Schaffers sign is seen in:


a. Glaucoma
b. Uveitisv
c. Retinal detachment
d. Keratitis
Ans-219: (c) Retinal detachment
Ref: Read the text below
Sol:
It is pigment dispersion from RPE in anterior vitreous due to retinal break. It is also called
tobacco dust.

Q-220 Treatment of moorens ulcer is :


a. Corneal graft
b. Immunosuppressives
c. Topical steroids
d. All of the above
Ans-220: (d) All of the above
Ref: Read the text below
Sol:
Mooren's ulcer is a rare peripheral ulcerative keratitis caused by ischemic necrosis
resulting from vasculitis of limbal vessels.
Apart from the above modality of treatment , we can also do conjunctival excision 3
mm from limbus parallel to the ulcer.
This is affective as conjunctiva adjacent to the ulcer has been found to secrete
collagenase and proteoglyconase.

Q-221 Sclera is thinnest at ?


a. Limbus
b. Insertion of rectus muscle
c. Equator
d. Posterior pole
Ans-221: (b) Insertion of rectus muscle
Ref: Read the text below
Sol:
Sclera is thinnest at lamina cribosa and then posterior to the attachment of the
muscles.

Q-222 Uveoparotitis is seen in :


a. Sarcoidosis
b. SLE
c. Scleroderma
d. Mumps

www.damsdelhi.com

Email: info@damsdelhi.com

DAMS CBT 2016 SET - 1

Ans-222: (a) Sarcoidosis


Ref: Read the text below
Sol:
Sarcoidosis causes granulomatous panuveitis with inflammation of parotid glands.
A rare manifestation of sarcoidosis is Heerfordts syndrome in which there is uveitis
with parotid inflammation , fever and sometimes associated with facial nerve palsy.

Q-223 Multifocal ERG is useful to assess the function of


a. Assessing rods
b. Assessing macular cones
c. Function of ganglion layer.
d. RPE
Ans-223: (b) Assessing macular cones
Ref: Read the text below
Sol:
Multifocal ERG allows the local ERG responses to be recorded from many regions of the
retina . An abnormal Multifocal ERG indicates that the foveal cones or bipolar cells are
dysfunctional and a source of vision loss . It also indicates precise distribution of retinal
dysfunction and can be correlated with field testing . It is particularly useful in
Unknown visual loss with normal appearing retina
Distinguishing between optic nerve and retinal disease
Diagnosis of focal cone dystrophy where mf ERG is the only diagnostic test .

Q-224 Antiglaucoma drug contraindicated in infants :


a. Brimonidine
b. Latanoprost
c. Bimatoprost
d. Timolol
Ans-224: (a) Brimonidine
Ref: Read the text below
Sol:
Brimonidine causes serious lethargy,hypotonia and CNS depression .

Q-225 Cupulliform cataract starts from :


a. Centre anteriorly
b. Centre posteriorly
c. Peripheral anteriorly
d. Peripheral posteriorly
Ans-225: (b) Centre posteriorly
Ref: Read the text below
Sol:

www.damsdelhi.com

Email: info@damsdelhi.com

DAMS CBT 2016 SET - 1

Cupulliform cataract is posterior subcapsular cataract and starts from centre


posteriorly
Q-226 Blood sugar estimation is relevant in all the following except:
a. Rh incompatibility
b. Birth asphyxia
c. SFD babies
d. Baby of hypothyroid mother
Ans-226: (d) Baby of hypothyroid mother
Ref: Read the text below
Sol :
High risk situations where screening for hypoglycemia is recommended:
1. Low birth weight infants (<2000 grams)
2. Preterm infants (35 weeks)
3. Small for gestational age infants (SGA) : birth weight <10th percentile
4. Infant of diabetic mothers (IDM) - insulin dependent and gestational diabetes
5. Large for gestational age (LGA) infants: birth weight >90th percentile*
6. Infants with Rh-hemolytic disease
7.Infants born to mothers receiving therapy with terbutaline/propranolol /labetolol/oral
hypoglycemic agents
8 Infants with IUGR
9. Any sick neonate such as those with perinatal asphyxia, polycythemia, sepsis, shock etc,
10. Infants on total parenteral nutrition

Q-227 The following may present as haemolytic disease of the newborn:


a. Hereditary spherocytosis
b. Sickle cell trait
c. Thalassaemia
d. None
Ans 227: (a) Hereditary spherocytosis
Reference: Read the text below
Sol:
HAEMOLYTIC DISEASE OF THE NEWBORN
Immune: (Rh, ABO, other)
Membrane defects: Spherocytosis, elliptocytosis
Enzyme defects: G6PD, PK, hexokinase
Sepsis
Polycythaemia: IDM, fetal transfusion.
Sickle cell and thalassaemia do not present in the neonatal period (HbF present).

Q-228 The following features are characteristic of William's Syndrome:


a. Short stature
b. Transient neonatal hypocalcaemia
c. Normal facies
www.damsdelhi.com

Email: info@damsdelhi.com

DAMS CBT 2016 SET - 1

d. Severe learning difficulties


Ans 228: (a) Short stature
Reference: Read the text below
Sol:
Recognized clinical features of William's Syndrome include:
1. Short stature
2. Characteristic facies "elfin" (full face with high rounded cheeks, broad forehead, flattened
bridge of the nose and long upper lip),
3. Idiopathic hypercalcaemia
4. Supravalvular aortic stenosis
5. Mild to moderate learning difficulties.

Q-229 Concerningblood flow in the fetus:


a. Blood flows fromleft to right through the foramen ovale.
b. The ductus arteriosus is closed.
c. Pulmonary pressure equals systemic pressure.
d. The haemoglobin may be 20g/dl.
Ans 229: (d) The haemoglobin may be 20g/dl.
Reference: Read the text below
Sol:
Persistence of the fetal circulatory pattern of right-to-left shunting through the patent
ductus arteriosus and foramen ovale after birth is due to an excessively highpulmonary
vascular resistance.
Fetal pulmonary vascular resistance is usually elevated relative to fetal systemic or
postnatal pulmonary pressure.
This fetal state permits shunting of oxygenated umbilical venous blood to the left atrium
(and brain) through the foramen ovale and bypasses the lungs through the ductus
arteriosus to the descending aorta.
After birth, pulmonary vascular resistance normally declines rapidly as a consequence
of vasodilatation due to gas filling the lungs, a rise in postnatal PaO2, a reduction in PCO2,
increased pH, and release of vasoactive substances.
Normal haemoglobin range in the first 1-3 days of life is between 14.5-22.5 g/dl.

Q-230 Sodium valproate is the one of the anticonvulsant medication for a 12-yr-old boy
with epilepsy.Which of the following factors markedly increases the risk of fatal liver
necrosis ?
a. Male gender
b. Age younger than 2 yr
c. Down syndrome
d. Family history of epilepsy
Ans 230: (b) Age younger than 2 yr
Reference: Read the text below
Sol:

www.damsdelhi.com

Email: info@damsdelhi.com

DAMS CBT 2016 SET - 1

Young age is a risk factor for hepatic and perhaps pancreatic complications of valproic
acid.
It is much safer in 12-yr-old children than those younger than 2 yr.

Q-231 A 10-yr-old girl is being evaluated for new onset of school problems, obsessivecompulsive behavior, and occasional uncontrolled movements of the hands. She has been
healthy and has not taken any medications. Further evaluation is most likely to reveal:
a. Brain tumor in the posterior fossa
b. Partial complex epilepsy
c. Evidence of streptococcal infection
d. Hydrocephalus
Ans 231: (c) Evidence of streptococcal infection
Reference: Read the text below
Sol:
Poststreptococcal obsessive-compulsive disorder is a well-recognized clinical entity.

Q-232 All are causes of IUGR excepta. Anemia


b. PIH
c. GDM
d. Maternal heart disease
Ans-232: (c) GDM
Ref: Read the text below
Sol :
Gestational DM causes MACROSOMIA; not IUGR

Q-233 Placenta is not as efficient as lungs. The umbilical venous pO2 is


a. 20-25 mm Hg
b. 25-28 mm Hg
c. 30-32 mm Hg
d. 40-45 mm Hg
Ans-233: (c) 30-32 mm Hg
Ref: Read the text below
Sol :
The placenta is not as efficient an oxygen exchange organ as the lungs, so that umbilical
venous Po2 (the highest level of oxygen provided to the fetus) is only about 30-35 mm Hg.
Approximately 50% of the umbilical venous blood enters the hepatic circulation,
whereas the rest bypasses the liver and joins the inferior vena cava via the ductus venosus,
where it partially mixes with poorly oxygenated inferior vena cava blood derived from the
lower part of the fetal body.
This combined lower body plus umbilical venous blood flow (Po2 of 26-28 mm Hg)
enters the right atrium and is preferentially directed by a flap of tissue at the right atrial

www.damsdelhi.com

Email: info@damsdelhi.com

DAMS CBT 2016 SET - 1

inferior vena caval junction, the eustachian valve, across the foramen ovale to the left
atrium.
This is the major source of left ventricular blood flow, since pulmonary venous return is
minimal.
Left ventricular blood is then ejected into the ascending aorta where it supplies
predominantly the fetal upper body and brain.

Q-234 The newborn cardiac output is


a. 200 ml/kg/min
b. 250 ml/kg/min
c. 300 ml/kg/min
d. 350 mL/kg/min
Ans-234 : (d) 350 mL/kg/min
Ref: Read the text below
Sol :
The newborn cardiac output (about 350 mL/kg/min) falls in the 1st 2 mo of life to about
150 mL/kg/min and then more gradually to the normal adult cardiac output of about 75
mL/kg/min

Q-235 New born baby are prone to hypoglycemia because ?


a. Low body glycogen reserve
b. Impaired glucose metabolism
c. Prematurity
d. All of the above
Ans-235: (a) Low body glycogen reserve
Ref: Read the text below
Sol :
New born babies are more prone to hypoglycemia because they have poor glycogen
stores.
The other options are causes of hypoglycemia in a new born and not why they can
normally be predisposed to it.

Q-236 Most common cause of neonatal meningitis is due to ?


a. Streptococcus group A
b. Streptococcus group B
c. Streptococcus group C
d. Streptococcus group D
Ans-236: (b) Streptococcus group B
Ref: Read the text below
Sol :
Group B Streptococcus is the most common cause of neonatal meningitis.

www.damsdelhi.com

Email: info@damsdelhi.com

DAMS CBT 2016 SET - 1

Second most common cause is E. coli.

Q-237 Most common bacterial cause of diarrhoea in children is?


a. E. coli
b. Salmonella
c. Staphylococcus
d. Shigella
Ans-237 : (a) E. coli
Ref: Read the text below
Sol :
Most common bacterial cause of diarrhea is ETEC (Enterotoxic E. coli).
Other causes :
Salmonella is most common cause of inflammatory diarrhea in children
Shigella, Cholera, Campylobacter are other common bacteria involved

Q-238 Best method of diagnosis of childhood HIV ?


a. CD4 cell counts
b. P24 antigen
c. HIV PCR
d. Anti HIVA antibody
Ans-238: (c) HIV PCR
Ref: Read the text below
Sol :
HIV DNA PCR is the preferred virologic assay in developed countries.
Almost 40% of infected newborns have positive test results in the 1st 2 days of life, with
>90% testing positive by 2 wk of age.
Plasma HIV RNA assays, which detect viral replication, may be more sensitive than DNA
PCR for early diagnosis, but data are limited.
The p24 antigen assay is also highly specific and essay to perform, but it is less sensitive
than the other virologic tests. It is not recommended for diagnosis of infection infants <1
months of age.

Q-239 Perioral rash & pigmentation of palmar area, diagnosis is?


a. Zn deficiency
b. Cu deficiency
c. Mg deficiency
d. Iron Deficiency
Ans-239: (a) Zn deficiency
Ref: Read the text below
Sol :
Perioral rash is very typical of zinc deficiency .

www.damsdelhi.com

Email: info@damsdelhi.com

DAMS CBT 2016 SET - 1

Although its association with palmar erythema couldnt be found, but the clinical
picture given most likely indicates zinc deficiency.

Q-240 Hyaline membrane is seen in all of the following conditions, except :


a. Radiation pneumonitis
b. Viral pneumonitis
c. Uremic pneumonitis
d. Staphylococcal bronchopneumonia
Ans-240: (d) Staphylococcal bronchopneumonia
Ref: Read the text below
Sol :
Hyaline membranes are seen in
ARDS or ACUTE lung injury
Hyaline membrane disease
Cyanotic congenital heart disease (TAPVC)
Early onset sepsis
Pneumonia
Maternal group B streptococcal infection
Meconium aspiration syndrome
Cystic adenomatoid malformation
Uremic pneumonitis
Radiation pneumonitis
Viral pneumonia

Q-241 Vertical transmission of Toxoplasmosis causes more severe disease in?


a. During labour
b. 1st trimester
c. 2nd trimester
d. 3rd trimester
Ans-241 : (b) 1st trimester
Ref: Read the text below
Sol :
When a mother acquires infection during gestation, organisms may disseminate
hematogenously to the placenta.
Infection may be transmitted to the fetus transplacentally or during vaginal delivery.
Of untreated maternal infections acquired in the 1st trimester, approximately 17% of
fetuses are infected, usually with severe disease.
Of untreated maternal infection acquired in the 3rd trimester, approximately 65% of
fetuses are infected, usually with disease that is mild or inapparent at birth
These different rates of transmission and outcomes are most likely related to placental
blood flow, virulence, inoculum of T. gondii, and immunologic capacity of the mother to
limit parasitemia.

www.damsdelhi.com

Email: info@damsdelhi.com

DAMS CBT 2016 SET - 1

Q-242 Which should be measured in a newborn who presents with hyperbilirubinemia?


a. Total and direct bilirubin
b. Total bilirubin only
c. Direct bilirubin only
d. Conjugated bilirubin only
Ans-242: (b) Total bilirubin only
Ref: Read the text below
Sol :
A child with Neonatal jaundice is evaluated and managed on the basis of Total serum
bilirubin only

Q-243 A baby presents with limb hypoplasia with scarring. What is the intra uterine
infection which can manifest in this way?
a. Varicella
b. Rubella
c. CMV
d. Toxoplasmosis
Ans-243: (a) Varicella
Ref: Read the text below
Sol :
Congenital Varicella classically presents with IUGR, Cicatrical(scarring) skin lesions in a
characteristic dermatomal distribution & limb hypoplasia

Q-244 Complete head control, without any head lag when pulled to sit, disappearance of
hand regard, bidextrous grasp able to sit with support all occur at about
a. 12 weeks
b. 16 weeks
c. 20 weeks
d. 24 weeks s
Ans-244 : (c) 20 weeks
Ref: Read the text below
Sol :
DEVELOPMENT MILESTONES (QUICK REVIEW):
Gross Motor:
3 months
Head holding
5 months
Sitting with support
6 months
Prone to supine, supine to prone
8 months
Sitting without support
9 months
Crawling
10 months
Creeping
1 year
Walking few steps independently
2 yr
walk up stairs with two feet at each step

www.damsdelhi.com

Email: info@damsdelhi.com

DAMS CBT 2016 SET - 1

3 yr
4 yrs
5 yr
Fine motor
12 weeks
4 months
5 months
7 months
9 months
13 months
15 months
18 months
24 months
3 yrs
4 yrs
5 yrs
Social
2 months
3 months
6-7 months
9 months

15 months
18 months
2 yrs
3 yrs
Language
1 month
3 months
6 months
9 months
1 yr
18 months
2 yrs
3 yrs
4 yrs
5 yrs

Up stairs with one foot at each step, rides tricycle


Hops on one foot
Skips on two feet

Grasp reflex disappears


goes for object
bidextrous grasp
Transfer object, palmar grasp
Pincer grasp
Casting
Self feed with a spoon
Self feed with a cup
tower of 4 cubes
turns pages of a book one at time
6-7 cubes
makes circle, tower of 10 cubes
Square, + sign
Triangle x sign

Social smile
recognizes mother
smiles at mirror image
waves bye-bye
Stranger anxiety
Peek a boo
hugs parents
kisses parents
Dry by day
dry by night
Dress/undress himself
head turns to sound
cooing
mono syllabus (ma,ba)
Bisyllabus (ma-ma, baba)
2 words with meaning
vocabulary of ten words
simple sentences with 2 words
tells age and sex, uses pronouns,handedness
tells story
knows colors

www.damsdelhi.com

Email: info@damsdelhi.com

DAMS CBT 2016 SET - 1

Q-245 Which among the following is not the major criteriain jones criteria for rhematic
fevera. Carditis
b. Chorea
c. Arthritis
d. Increased E.S.R.
Ans-245: (d) Increased E.S.R.
Ref: Read the text below
Sol :
Revised Jones criteria for diagnosis of Acute Rheumatic fever
Major criteria
Minor criteria
* Carditis
* Migratory polyarthritis
* Erythema marginatum

* Fever
* Arthralgia
* Elevated acute phase reactants ( ESR, CRP
)
* Prolonged PR interval

* Chorea
* Subcutaneous nodules
Essential criteria: Evidence of a preceding group A streptococcal infection ( culture, rapid
antigen, antibody rise/ elevation )

Q-246 A diabetic female on INH and rifampin for TB suffers DVT: She is started on
warfarin: PT is not raised; next step should be:
a. Long term heparin therapy
b. Replace warfarin with acecoumarin
c. Switch ethambutol for rifampin
d. Use LMW heparin.
Ans-246: (c) Switch ethambutol for rifampin
Ref:Read the text below
Sol:
Rifampin is a microsomal enzyme inducer enhances its own metabolism as well as
that of many drugs including warfarin, oral contraceptives, corticosteroids sulfonylureas,
digitoxin, HIV protease inhibitors, ketoconazole etc.

Q-247 Secukinumab is a new drug that is used to treat Ankylosing Spondylitis.which of the
following cytokines is inhibited by this drug?
a. Interleukin 1
b. TNF alpha
c. Interleukin 17 A
d. Interleukin 6
Ans-247: (c) Interleukin 17 A
Ref: Read the text below
Sol:
www.damsdelhi.com

Email: info@damsdelhi.com

DAMS CBT 2016 SET - 1

Secukinumab is a monoclonal antibody that binds to Interleukin 17 A


The drug was first approved for the treatment of chronic plaque Psoriasis.
The drug is now approved for the treatment of Psoriatic arthritis and Ankylosing
spondylitis also.

Q-248 The Bruton Tyrosine Kinase inhibitor, Ibrutinib is used to treat which of the
following hematological malignancy?
a. Chronic myelogenous leukemia
b. Acute myeloid leukemia
c. Chronic Lymphocytic Leukemia
d. Multiple Myeloma
Ans-248: (c) Chronic Lymphocytic Leukemia
Ref: Read the text below
Sol:
Ibrutininb is an orally administered covalent inhibitor of Bruton Tyrosine Kinase.It has been
approved for the treatment of Chronic lymphocytic leukemia,mantle cell lymphoma and
Waldenstroms macroglobulinemia.
Newer molecules for treatment of CLL:
Ibrutinib : Bruton Tyrosine Kinase inhibitor
Idelalisib: Phosphoinositide 3-kinase delta inhibitor
Ofatumumab,Obinutuzumab : Anti CD20 monoclonal antibodies

Q-249 All of the following drugs are recommended for treatment of beta blocker induced
excessive bradycardia and/or decrease in cardiac output except :a. Dopamine
b. Dobutamine
c. Glucagon
d. Phentolamine
Ans-249: (d) Phentolamine
Ref:Read the text below
Sol:
Drugs useful in b blocker toxicity/Bradycardia:
Atropine
Isproterenol
Glucagon
Dopamine & Dobutamine
Epinephrine/norepinephrine

Q-250 A patient has completed a course of cancer chemotherapy and now has severe
anemia, neutropenia, and thrombocytopenia. If only one intervention is possible, which of
the following is the most appropriate therapy?
a. Epoetin

www.damsdelhi.com

Email: info@damsdelhi.com

DAMS CBT 2016 SET - 1

b. Filgrastim
c. Growth hormone
d. Sargramostim
Ans-250: (d) Sargramostim
Ref:Read the text below
Sol:
Apatient who is anemic, neutropenic, and thrombocytopenic requires stimulation of all
three major cell lines in the bone marrow.
The only drug currently available that accomplishes this broad-spectrum stimulant
effect is sargramostim (granulocytemacrophage colony stimulating factor [GM-CSF]).
Epoetin is a more selective stimulant of erythrocyte production and is useful in simple
anemia.
Filgrastim is a somewhat selective stimulant of leukocyte production and has much less
effect on erythrocytes and platelet production than sargramostim
Growth hormone and testosterone have both been tried in the treatment of anemia
with negligible success.

Q-251 Which of the following is the agent of choice for chronic treatment of simple
hypothyroidism (myxedema)?
a. Desiccated thyroid
b. Levothyroxine (T4)
c. Liothyronine (T3)
d. Potassium iodide
Ans-251: (b) Levothyroxine (T4)
Ref:Read the text below
Sol:
The agent of choice in treating simple hypothyroidism is levothyroxine (T4).
Administered thyroxine is bound in plasma in the same way and metabolized to triiodothyronine in the same manner as native thyroxine.
Although liothyronine (tri-iodothyronine [T3]) ismore active than T4 as an agonist at
thyroid hormone receptors, its shorter half-life of 1 day as compared with the 67-day halflife of T4 means that thyroxine has a longer duration of action and provides smoother
control.

Q-252 Which of the following correctly describes the mechanism of action of


streptokinase?
a. Combines with plasminogen to form an enzymatically active complex
b. Competitively Blocks Binding Of Plasminogen To Fibrin
c. Converts plasmin to plasminogen
d. Inhibits platelet cyclooxygenase (COX) activity
Ans-252: (a) Combines with plasminogen to form an enzymatically active complex
Ref:Read the text below
Sol:

www.damsdelhi.com

Email: info@damsdelhi.com

DAMS CBT 2016 SET - 1

Streptokinase has no intrinsic enzymatic activity, but instead forms a stable complex
with the patients plasminogen, making it enzymatically active in cleaving free
plasminogen to plasmin.
The streptokinase plasminogen complex is not inhibited by antiplasmin. The other
thrombolytic agentst-PA, reteplase, tenecteplase, and urokinase activate plasminogen
directly.

Q-253 A17-year-old patient suffers from tonicclonic seizures. This condition has been
well controlled with a regimen of phenytoin. Which of the following signs or symptoms
indicates phenytoin toxicity?
a. Diplopia and abnormal gait
b. Hyperprolactinemia
c. Polydipsia and polyuria
d. Postural hypotension
Ans-253: (a) Diplopia and abnormal gait
Ref:Read the text below
Sol:
Diplopia, abnormal gait, and other signs of cerebellar dysfunction are important
symptoms of phenytoin toxicity.
Other manifestations of toxicity include gingival hyperplasia, nystagmus,and vertigo.
Hyperprolactinemia is an adverse effect of antipsychotic dopamine antagonists such as
the phenothiazines; dopamine inhibits prolactin secretion by the anterior pituitary.
Polydipsia and polyuria are symptoms of diabetes insipidus.
These symptoms may be produced by lithium toxicity during treatment of bipolar
depression, and are not associated with phenytoin toxicity.
Postural hypotension is not an adverse effect of phenytoin but often occurs with
levodopa treatment of Parkinsons disease.

Q-254 Which of the following substances contains the highest concentration of molecules
with estrogenic effects?
a. Ginseng
b. Grapefruit juice
c. Saw palmetto
d. Tomatoes
Ans-254: (c) Saw palmetto
Ref:Read the text below
Sol:
Saw palmetto is promoted by purveyors of herbs and alternative medicines as
therapy for prostatic hyperplasia and does have weak estrogenic activity.
Grapefruit juice and tomatoes are used therapeutically in a treatment regimen referred
to as raw juice therapy.

www.damsdelhi.com

Email: info@damsdelhi.com

DAMS CBT 2016 SET - 1

Q-255 Selective serotonin Reuptake Inhibitor (SSRI) are all except:


a. Fluphenazine
b. Fluoxetine
c. Fluvoxamine
d. Sertraline
Ans-255: (a) Fluphenazine
Ref: CMDT Pg. 1091
Sol :
Selective serotonin reuptake inhibitors are :
Fluoxetine
Fluvoxamine
Paroxetine
Duloxetine
Sertraline
Citalopram
Fluphenazine and fluphenthixol are antipsychotics.

Q-256 Megaloblastic anemia is caused by all except


a. Aspirin
b. Primidone
c. Methotrexate
d. Triamterene
Ans-256: (a) Aspirin
Ref: Harrisons - 433
Sol :
Megaloblastic anemia is caused by certain drugs such as
Co-trimoxazole - Folate antagonists (Mtx)
N 2O
- OCP
Phenobarbital - Phenytoin
Primidone
- Triamterene
Aspirin causes hemolytic anemia in G6PD deficiency.
Chloramphenicol causes aplastic anemia by causing BM suppression.

Q-257 Thromboprophylactic agent is


a. Dextran
b. Gelatin
c. Haemacil
d. Diethyl starch
Ans-257: (a) Dextran
Ref: KDT - 584
Sol :
Dextran 40 has been tried in stroke and for prophylaxis of deep vein thrombosis (i.e.
thromboprophylactic) and pulmonary infarction.

www.damsdelhi.com

Email: info@damsdelhi.com

DAMS CBT 2016 SET - 1

Q-258 Exenatide is a newer drug proposed to be used in the treatment of


a. Osteoporosis
b. Diabetes mellitus
c. Hyperparathyroidism
d. Anovulatory infertility
Ans-258: (b) Diabetes mellitus
Ref: Read the text below
Sol :
Exenatide is a glucagon like peptide analogue which is proposed to be used in the
treatment of post prandial hyperglycemia.
Exenatide is believed to facilitate glucose control in at least five ways:
1. Exenatide augments pancreas response (i.e. increases insulin secretion) in response to
eating meals; the result is the release of a higher, more appropriate amount of insulin that
helps lower the rise in blood sugar from eating. Once blood sugar levels decrease closer to
normal values, the pancreas response to produce insulin is reduced; however, other drugs
(like injectable insulin) are effective at lowering blood sugar, but can "overshoot" their
target and cause blood sugar to become too low, resulting in the dangerous condition of
hypoglycemia.
2. Exenatide also suppresses pancreatic release of glucagon in response to eating, which
helps stop the liver from overproducing sugar when it is unneeded, which prevents
hyperglycemia (high blood sugar levels).
3. Exenatide helps slow down gastric emptying and thus decreases the rate at which
meal-derived glucose appears in the bloodstream.
4. Exenatide has a subtle yet prolonged effect to reduce appetite, promote satiety via
hypothalamic receptors (different receptors than for amylin). Most people using exenatide
slowly lose weight, and generally the greatest weight loss is achieved by people who are the
most overweight at the beginning of exenatide therapy. Clinical trials have demonstrated
the weight reducing effect continues at the same rate through 2.25 years of continued
use. When separated into weight loss quartiles, the highest 25% experience substantial
weight loss, and the lowest 25% experience no loss or small weight gain.
5. Exenatide reduces liver fat content. Fat accumulation in the liver or nonalcoholic fatty
liver disease (NAFLD) is strongly related with several metabolic disorders, in particular low
HDL cholesterol and high triglycerides, present in patients with type 2 diabetes. It became
apparent that exenatide reduced liver fat in miceand more recently in man.

Q-259 A drug X belongs to the anticholinergic drug group. It is primarily used in pre
anesthetic medication and also during surgery. Which of the following can be X ?
a. Glycopyrrolate
b. Pipenzolate methyl bromide
c. Isopropamide
d. Dicyclomine
Ans-259: (a) Glycopyrrolate
Ref: Read the text below
Sol :
www.damsdelhi.com

Email: info@damsdelhi.com

DAMS CBT 2016 SET - 1

Glycopyrrolate is used to reduce the secretions (to prevent reflex bronchospasm)


during anaesthesia.
In anesthesia, glycopyrrolate injection can be used as a preoperative medication in
order to reduce salivary, tracheobronchial, and pharyngeal secretions, as well as
decreasing the acidity of gastric secretion.
It is also used in conjunction with neostigmine, a neuromuscular blocking reversal
agent, to prevent neostigmine's muscarinic effects such as bradycardia.
Q-260 All of the following mydriatics produce cycloplegia except
a. Phenylephrine
b. Tropicamide
c. Cyclopentolate
d. Homatropine
Ans-260: (a) Phenylephrine
Ref: KDTs - 113, 114
Sol :
Anticholinergic agents produce both cycloplegia as well as mydriasis whereas
sympathomimetics cause mydriasis without inducing cycloplegia.

Q-261 Which of the following drugs shows the phenomenon of vasomotor reversal of Dale
after administration of an adrenergic blocker ?
a. Adrenaline
b. Noradrenaline
c. Isoprenaline
d. All of the above
Ans-261: (a) Adrenaline
Ref: KDTs - 123
Sol :
Vasomotor reversal of dale is seen with adrenaline.
When this drug is infused quickly, initially there is rise in blood pressure (due to
sitmulation of receptors) followed by prolonged fall (2 action). blocking drugs
inhibit the initial rise and only fall in blood pressure is recorded.
This is known as vasomotor reversal.
Nor adrenaline has no 2 activity and isoprenaline lacks activity, therefore cannot
demonstrate this phenomenon.
Q-262 A female patient suffering from psychosis is taking phenothiazines. She now
complains of sudden onset of high-grade fever, muscle rigidity and altered sensorium. The
most likely diagnosis is
a. Malignant hyperthermia
b. Neuroleptic malignant syndrome
c. Tardive dyskinesia
d. Akathesia
Ans-262: (b) Neuroleptic malignant syndrome
Ref: Read the text below
www.damsdelhi.com

Email: info@damsdelhi.com

DAMS CBT 2016 SET - 1

Sol :
Neuroleptic malignant syndrome has developed in this patient.
It may occur with high doses of phenothiazines. It lasts 5 10 days after drug
withdrawal and may be fatal.
The neuroleptic must be stopped promptly and symptomatic treatment should be
given.
Dantrolene and bromocriptine in large doses have been found to be useful.
Antiparkinsonian anticholinergics are of no help in this situation.

Q-263 The major difference between typical and atypical antipsychotics is that :
a. The latter cause minimal or no increase in prolactin
b. The former cause tardive dyskinesia
c. The former are available as parenteral preparations
d. The latter cause substantial sedation
Ans-263: (b) The former cause tardive dyskinesia
Ref: Read the text below
Sol :
The most important reason to classify the antipsychotics in typical and atypical agents
is the occurrence of extrapyramidal symptoms.
Typical antipsychotics have significant extrapyramidal symptoms such as parkinsonism,
acute muscle dystonia and tardive dyskinesia. These side-effects are negligible or minimal
with atypical agents like clozapine, olanzapine and risperidone.

Q-264 Antipyschotic drug-induced Parkinsonism is treated by :


a. Anticholinergics
b. Levodopa
c. Selegiline
d. Amantadine
Ans-264: (a) Anticholinergics
Ref: Read the text below
Sol :
For antipsychotic drug induced parkinsonism, anticholinergics are the drug of choice
e.g. Trihexphenidyl.
Drugs facilitating dopaminergic system (e.g. Levodopa, selegiline and Amantadine) are
not much effective as the dopamine receptors are already blocked by antipsychotics
(antidopaminergic) in the drug induced vomiting.
Q-265 In a log dose response curve, which of the following is a false statement about drug
efficacy and potency
a. Efficacy of drug is more important than potency
b. ED50 denotes efficacy
c. Higher peak mean higher efficacy
d. Two drugs having same efficacy can have different pharmacological profile
Ans-265: (b) ED50 denotes efficacy
www.damsdelhi.com

Email: info@damsdelhi.com

DAMS CBT 2016 SET - 1

Ref: Read the text below


Sol :
ED50 is the dose of a drug producing 50% of the maximal effect. So, it indicates potency
of the drug.
Higher upper limit of log dose response curve (log DRC) means higher efficacy.
Efficacy of a drug is more important factor than potency in making choice of the drug.
Two drugs can have same efficacy e.g. analgesic efficacy but may have different
pharmacological effects and side effects.

Q-266 Which among the following agents is the most common cause of Prosthetic valve
endocarditis arising within one year of surgery?
a. Staphylococcus aureus
b. Streptococcus viridans
c. Coagulase negative staphylococcus
d. Streptococcus bovis
Ans-266: (c) Coagulase negative staphylococcus
Ref: Read the text below
Sol:
In the first one year the most common cause of Prosthetic valve endocarditis is
Coagulase negative staphylococcus.
After one year the most common organism causing prosthetic valve infection is
Streptococcus viridans
Most common cause of community acquired native valve endocarditis : Streptococcus
viridians
Most common cause of health careassociated Native valve endocarditis
:Staphylococcus aureus
Most common cause Injection drug useassociated endocarditis : Staphylococcus
aureus
Etiology of endocarditis that occurs in association with colonic polyps and tumors :
Streptococcus gallolyticus (previously called S.bovis)

Q-267 A 35-year-old dialysis patient presents to the emergency department complaining


of being unable to move.He had missed his dialysis treatment the previous day because of
weakness. An electrocardiogram shows peaked T waves,Next Step
a. Schedule emergent dialysis
b. Give a sodium polystyrene sulfonate retention enema
c. Administer insulin and glucose
d. Administer calcium gluconate
Ans-267: (d) Administer calcium gluconate
Ref: Read the text below
Sol :

www.damsdelhi.com

Email: info@damsdelhi.com

DAMS CBT 2016 SET - 1

Peaked T waves are a sign of hyperkalemia, and the patients history of missed dialysis
treatment and profound weakness is consistent with the diagnosis.
Given the changes in the ECG, immediate therapy is needed.
The aim of therapy is to antagonize the membrane effects of the hyperkalemia and
effect potassium removal from the body.
The physician should not wait for confirmatory laboratory work in this setting. Calcium
gluconate has its onset of action within minutes and would be the first line of therapy in this
patient. Insulin and glucose are effective at causing a shift of potassium into the
intracellular space, but onset of action is 20 to 30 minutes. Sodium polystyrene sulfonate is
effective at removing potassium, but it takes 30 to 60 minutes to be effective, and there
may be a similar delay in getting dialysis started. An inhaled B- against and NaHCO3, are
also effective , but they take effect more slowly than calcium.

Q-268 All of the following conditions may be seen as an expression of myelomaassociated renal disease except
a. Chronic tubulointerstitial nephropathy
b. Acute renal failure (ARF)
c. Isolated functional tubular defects
d. Membranoproliferative glomerulonephritis
Ans-268: (d) Membranoproliferative glomerulonephritis
Ref: Read the text below
Sol :
Renal insuffiency or renal failure can occur in 35% to 50% of patients with multiple
myeloma (MM) and can have variable expressions.
Renal failure can be classified in one of four general categories, including chronic
tubulointerstitial nephropathy, acute renal failure, chronic glomerulopathy and isolated
functional tubular defects.
It is believed that Bence Jones proteinuria plays an important role in each of these
syndromes.

Q-269 A 29-year-old woman is in the intensive care unit with rhabdomyolysis due to
compartment syndrome of the lower extremities after a car accident. Her clinical course
has been complicated by acute renal failure and severe pain. She has undergone
fasciotomies and is admitted to the intensive care unit. An electrocardiogram (ECG) is
obtained (shown below). What is the most appropriate course of action at this point?

www.damsdelhi.com

Email: info@damsdelhi.com

DAMS CBT 2016 SET - 1

a. 18-lead ECG
b. Coronary catheterization
c. Hemodialysis
d. Intravenous fluids and a loop diuretic
Ans-269: (d) Intravenous fluids and a loop diuretic
Ref: Read the text below
Sol:
This ECG shows a short ST segment that is most prominent in V2, V3, V4, and V5.
Hypercalcemia, by shortening the duration of repolarization, abbreviates the total time
from depolarization through repolarization.
This is manifested on the surface ECG by a short QT interval. In this scenario, the
hypercalcemia is due to the rhabdomyolysis and renal failure. Fluids and a loop diuretic are
an appropriate therapy for hypercalcemia.
Hemodialysis is seldom indicated. Hemodialysis is indicated for significant
hyperkalemia, which may also develop after rhabdomyolysis, manifest by tenting of the T
waves or widening of the QRS.
Classic ECG manifestations of a pulmonary embolus (S1, Q3, T3 pattern) are infrequent
in patients with pulmonary embolism (PE), though the changes may be seen with massive
PE.
There are no signs of myocardial ischemia on this ECG, which would make coronary
catheterization and 18- lead ECG interpretation of low yield.

Q-270 Which of the following is the most appropriate therapy for a 60-year-old male with 2
weeks of productive cough, fever, shortness of breath, and the chest radiogram as shown in
the following figure?

www.damsdelhi.com

Email: info@damsdelhi.com

DAMS CBT 2016 SET - 1

a. Cephalexin
b. Ciprofloxacin
c. Clindamycin
d. Penicillin
Ans-270: (c) Clindamycin
Ref: Read the text below
Sol:
The radiograph describes a lung abscess that most likely is due to anaerobic infection.
The anaerobes involved are most likely oral, but Bacteroides fragilis is isolated in up to 10%
of cases.
Vancomycin, ciprofloxacin, and cephalexin have no significant activity against
anaerobes.

-lactamase. For many years


penicillin was considered the standard treatment for anaerobic lung infections.
However, clinical studies have demonstrated the superiority of clindamycin over
penicillin in the treatment of lung abscess. When there are contraindications to
clindamycin, penicillin plus metronidazole is likely to be as effective as clindamycin.

Q-271 In erectile dysfunction, which of the following drug is not used?


a. Apomorphine
b. Phenylephrine
c. Sildenafil
d. Papaverine
Ans-271: (b) Phenylephrine
Ref:Read the text below
Sol:
www.damsdelhi.com

Email: info@damsdelhi.com

DAMS CBT 2016 SET - 1

List of drugs causing erectile dysfunction


Classification

Drugs

Diuretics

Thiazides
Spironolactone

Antihypertensives

Calcium channel blockers


Methyldopa
Clonidine
Reserpine
-Blockers
Guanethidine

Cardiac/anti-hyperlipidemics Digoxin
Gemfibrozil
Clofibrate
Antidepressants

Selective serotonin reuptake inhibitors


Tricyclic antidepressants
Lithium
Monoamine oxidase inhibitors

Tranquilizers

Butyrophenones
Phenothiazines

H2 antagonists

Ranitidine
Cimetidine

Hormones

Progesterone
Estrogens
Corticosteroids
GnRH agonists
5 -Reductase inhibitors
Cyproterone acetate

Cytotoxic agents

Cyclophosphamide
Methotrexate
Roferon-A

Anticholinergics

Disopyramide
Anticonvulsants

Recreational

Ethanol
Cocaine
Marijuana

Q-272 A73-year-old patient has heart failure that has worsened over the past 12 months.
Which of the following drugs is likely to slow progression even though it has no direct
positive or negative effects on cardiac contractility?
a. Digoxin
www.damsdelhi.com

Email: info@damsdelhi.com

DAMS CBT 2016 SET - 1

b. Dobutamine
c. Losartan
d. Nesiritide
Ans-272: (c) Losartan
Ref:Read the text below
Sol:
Losartan, an AT1 angiotensin receptor antagonist, has been shown to slow the
progression of heart failure, in common with the ACE inhibitors, beta blockers, and
spironolactone, which also slow progression of this disease.
Positive inotropic agents such as digoxin and dobutamine do not slow progression
although they may be useful for reducing symptoms.
Nesiritide is a peptide that causes vasodilation and sodium diuresis and may have
value in acute but not chronic failure.

Q-273 Measurement of which urinary electrolyte is particularly useful in the diagnosis of a


patient with metabolic alkalosis?
a. Urine urea
b. Urine Na
c. Urine Cl
d. Urine volume
Ans-273: (c) Urine Cl
Ref: Read the text below
Sol :
Measurement of the urinary chloride (UCL) is a particularly useful parameter in gauging
the volume status of patients with metabolic alkalosis, and is used to classify patients with
metabolic alkalosis as either volume repleted (UCL >30 m Eq/L) or volume depleted (UCL
< 30 m Eq/L).
This allows determination of a differential diagnosis and guides the course of
treatment. In cases of metabolic alkalosis,theurine sodium is not a good indicator of
volume status, because the HCO3 lost in the urine forces with it a certain amount of Na.

Q-274 A 47 year old HIV patient presents with a white,frond like lesions on the lateral
border of the tongue as shown.Which of the following statements regarding the lesion is
not true ?

www.damsdelhi.com

Email: info@damsdelhi.com

DAMS CBT 2016 SET - 1

a. The etiological agent is Epstein-Barr Virus


b. The lesion is premalignant
c. Lesions may respond to Acyclovir
d. It is a WHO stage III disease in HIV infection
Ans-274: (b) The lesion is premalignant
Ref: Read the text below
Sol:
The image shows the classical appearance of Oral Hairy Leucoplakia.
Oral hairy Leucoplakia
The etiological agent of OHL is considered to be Epstein-Barr virus
OHL lesions appear to be relatively specific for HIV infection, since they are only rarely
observed in patients with other immunodeficiencies
OHL is not considered a premalignant lesion, being unlikely to progress to squamous
cell carcinoma
Severe cases have been reported to respond to topical podophyllin or systemic therapy
with anti-herpesvirus agents.
Q-275 All the following statements regarding Daptomycin are true except:
a. It is active against VRSA
b. It acts by depolarization of the cell membrane and Potassium efflux
c. Can be used in pneumonia caused by MRSA
d. Dose reduction is needed in patients with renal failure
Ans-275: (c) Can be used in pneumonia caused by MRSA
Ref: Read the text below
Sol:

www.damsdelhi.com

Email: info@damsdelhi.com

DAMS CBT 2016 SET - 1

Pulmonary surfactant antagonizes the action of Daptomycin, and therefore it should not
be used to treat pneumonia
Daptomycin :
Daptomycin is a novel cyclic lipopeptide antibiotic
This drug is active against staphylococci (including MRSA and coagulase-negative
staphylococci), streptococci, and enterococci. Daptomycin remains active against
Staphylococcus aureus and enterococci that are resistant to vancomycin.
Mechanism of action :
It bind to the cell membrane via calcium-dependent insertion of its lipid tail.
This results in depolarization of the cell membrane with potassium efflux and rapid cell
death.
Daptomycin is cleared renally.
Dose modification is required if creatinine clearance of less than 30 mL/min.
Side effects : Myopathy,Allergic pneumonitis

Q-276 Which of the following agents causes non inflammatory diarrhea?


a. Salmonella enteritidis
b. Clostridum perfringens
c. Yersinia enterocolitica
d. Shigella flexneri
Ans-276: (b) Clostridum perfringens
Ref: Read the text below
Sol:
Clostridium perfringens causes a toxin mediated non inflammatory diarrhea , while all the
other three organisms causes invasive inflammatory diarrhea
Inflammatory Diarrhea
Non inflammatory
diarrhea
Mechanism
Invasion or cytotoxin mediated
Enterotoxin mediated
Site of intestine
Distal small bowel and colon
Proximal small bowel
affected
Clinical presentation Dysentery or inflammatory diarrhea
Watery diarrhea
Stool finding
Fecal polymorphonuclear leukocytes;
No fecal leukocytes; mild
substantial increase in fecal lactoferrin or no increase in fecal
lactoferrin
Examples
Shigella spp., Salmonella spp.,
Vibrio
Campylobacterjejuni,EHEC,EIEC,Yersinia cholerae,ETEC,EAEC,
enterocolitica, Listeria monocytogenes, Clostridium perfringens,
Vibrio parahaemolyticus, Clostridium
Bacillus cereus,
difficile,Entamoeba histolytica,
Staphylococcusaureus,
Klebsiella oxytoca
rotavirus, norovirus,
enteric adenoviruses,
Giardia lamblia,
Cryptosporidium spp.,
Cyclospora spp.,

www.damsdelhi.com

Email: info@damsdelhi.com

DAMS CBT 2016 SET - 1

microsporidia

Q-277 The most common valvular lesion seen in carcinoid is


a. Mitral regurgitation (MR) and aortic stenosis (AS)
b. Pulmonary stenosis (PS) and tricuspid regurgitation (TR)
c. Pulmonary stenosis and aortic stenosis
d. Mitral stenosis (MS) and aortic stenosis
Ans-277: (b) Pulmonary stenosis (PS) and tricuspid regurgitation (TR)
Ref: Read the text below
Sol :
The cardiac lesion seen in carcinoid heart disease may be related to large quantities of
circulating serotonin,bradykinin or other substances that are inactivated by the lung.
This deactivation spares the aortic and mitral valves from the downstream fibrous
plaques that form.
The most common hemodynamic lesion seen with carcinoid is tricuspid regurgitation
and occasional pulmonary stenosis.
Sixty to ninety percent of tumors arise from the appendix, and the bronchus. It is rare
for carcinoid heat disease to resultfrom lesions that have not invaded the liver.

Q-278 The presence of which of the following conditions portends the poorest longterm
prognosis in patients with aortic stenosis?
a. Angina
b. Syncope
c. Valve area of less than 0.5cm2
d. CHF
Ans-278: (d) CHF
Ref: Read the text below
Sol :
CHF has the poorest long-term prognosis at 2 years,while syncope and angina have a
prognosis of 3 and 5 years,respectively.
The valve area or calcification offer no information concerning prognosis.

Q-279 All the following are considered as Complicated UTI except:


a. Pyelonephritis in a nonpregnant woman
b. Cystitis in males
c. Catheter associated UTI
d. Cystitis in pregnant females
Ans-279: (a) Pyelonephritis in a nonpregnant women
Ref: Read the text below
Sol:

www.damsdelhi.com

Email: info@damsdelhi.com

DAMS CBT 2016 SET - 1

Uncomplicated UTIrefers to acute cystitis or pyelonephritis in nonpregnant outpatient


women without anatomic abnormalities or instrumentation of the urinary tract; the term
complicated UTI encompasses all other types of UTI

Q-280 Which of the following organisms causes chronic recurrent meningitis?


a. HSV 2
b. Mycobacterium tuberculosis
c. Treponema pallidum
d. Cryptococcus neoformans
Ans-280: (a) HSV 2
Ref: Read the text below
Sol:
Chronic meningitis is diagnosed when features of meningitis exists for >4 weeks and is
associated with a persistent inflammatory response in the cerebrospinal fluid (CSF) (white
blood cell count >5/L).
There are two clinical forms of chronic meningitis :
Chronic persistent meningitis
Chronic recurrent meningitis
In chronic recurrent meningitis there is recurrent, discrete episodes of illness.All the
symptoms, signs, and CSF parameters of meningeal inflammation resolve completely
between episodes without specific therapy.
Causes of chronic recurrent meningitis :
Herpes simplex virus (HSV) type 2
Chemical meningitis due to episodic leakage from an epidermoid tumor,
craniopharyngioma, or cholesteatoma into CSF
Primary autoimmune inflammatory conditions, including Vogt- Koyanagi-Harada
syndrome, Behcets syndrome, systemic lupus erythematosus (SLE), and Mollarets
meningitis; and drug hypersensitivity with repeated administration of the offending agent.

Q-281 The condition least likely to be associated with prolonged hyperamylasemia is


a. Pseudocyst
b. Macroamylasemia
c. Passage of a common duct stone
d. Pancreatic ascities
Ans-281: (c) Passage of a common duct stone
Ref: Read the text below
Sol :
Prolonged hyperamylasemia after a bout of acute pancreatitis usually signifies
continuing pancreatitis or pseudocyst or abscess,and may present with pancreatic ascites.
If a patients has macroamylasemia,the amylase is attached to a protein,usually a
globulin, and is renal clearance is diminished and circulating values may be high and
continue to be elevated after a course of acute pancreatitis.

www.damsdelhi.com

Email: info@damsdelhi.com

DAMS CBT 2016 SET - 1

Passage of a common duct stone is associated with pancreatitis, which generally


rapidly resolves after passage of stone.

Q-282 DMARDs that may cause hematologic toxicity include all of the following except
a. Aurothioglucose
b. Aurothiomalate
c. Azathioprine
d. Hydroxycholoroquine
Ans-282: (d) Hydroxycholoroquine
Ref: Read the text below
Sol :
The parenterally administered gold compound aurothioglucose and aurothiomalate
may cause blood dyscrasias, including panytopenia.
Azathioprine, a purine analogue that interfers with DNA synthesis, may cause bone
narrow suppression and carries a longterm risk of lymphoproliferative malignancy.
D-penicillamine, a metal chelate, causes neutropenia and thrombocytopenia.
Hydroxychloroquine is not known to be a hematologic toxin.

Q-283 The most common ophthalmologic manifestation of Reiters syndrome isa. Conjuctivitis
b. Episcleritis
c. Keratitis
d. Posterior uveitis
Ans-283: (a) Conjuctivitis
Ref: Read the text below
Sol :
Reiters syndrome is defined as the triad of arthritis, urethritis,and conjunctivitis.
Conjunctivitis is unilateral or bilateral , and it occurs in 40% of patients with Reiters
syndrome.
Anterior uveitis occurs less often but is more severe.
Posterior uveitisoccurs rarely , as do keratitis, corneal ulceration, and optic neuritis.
Scleritis and episcleritis occur in RA.

Q-284 Uniform swelling of the finger or toe(sausage digit) is characteristic of all of


the following except
a. Reiters syndrome
b. Ankylosing spondylitis
c. Psoriatic arthritis
d. Sarcoid arthropathy
Ans-284: (b) Ankylosing spondylitis
Ref: Read the text below
Sol :

www.damsdelhi.com

Email: info@damsdelhi.com

DAMS CBT 2016 SET - 1

The sausage digit results from inflammation at the sites of attachment of the tendons
of the intrinsic muscles onto the phalanges,rather than the inflammation of the synovial
joint ,which is characteristic of RA.
The sausage digit resulting from dactylitis is seen in Reiters syndrome, psoriatic
arthritis, and chronic sarcoid arthropathy.
Sausage digits are not a manifestation of ankylosing spondylitis.

Q-285 A 30- year old man is noted to be hypertensive. A complete blood count reveals
mild leukocytosis,hematocrit of 55% ,and thrombocytosis.Uric acid is elevated and
ultrasound confirms splenomegaly.Comment on the diagnosis:
a. Polycythemia vera
b. Spurious polycythemia
c. Second polycythemia
d. Essential thrombocytosis
Ans-285: (a) Polycythemia vera
Ref: Read the text below
Sol :
Patients with spurious polycythemia frequently are dehydrated and volume
contracted; with future hyderation, their hematocrit may indeed normalize.
There is usually no evidence of a true myeloproliferative disorder ,including
splenomegaly or extensive thrombocytosis.
Patients with secondary polycythemia frequently have underlining chronic obstructive
pulmonary disease and an elevation in red blood cell mass as a compensatory mechanism.
Patients with true polycythemia vera usually have significant splenomegaly, elevated
leukocyte alkaline phosphatase, and frequently also exhibit leukocytosis and
thrombocytosis.

Q-286 In a patient with catheter related blood stream infection all the following are
indication for removal of the catheter, except:
a. Hemodynamic instability
b. Persistent bacteremia after 72 hours of antimicrobial therapy
c. Isolation of Coagulase negative staphylococcus from the blood
d. Endocarditis
Ans-286: (c) Isolation of Coagulase negative staphylococcus from the blood
Ref: Read the text below
Sol:
In patients with catheter related blood stream infection the following are indication for
removal of the catheter :
Severe sepsis,
Hemodynamic instability,
Suppurative thrombophlebitis,
Endocarditis,

www.damsdelhi.com

Email: info@damsdelhi.com

DAMS CBT 2016 SET - 1

Persistent bacteremia after 72 hours of antimicrobial therapy to which the organism is


susceptible
Infection due to S. aureus, P. aeruginosa, fungi or mycobacteria.

Q-287 An 80 year old man presents to the emergency department with complaints of
fever and headache for 5 days duration. On examination the patient is drowsy and has neck
stiffness. Which of the following is the preferred empirical therapy for suspected
meningitis in this patient?
a. Vancomycin and ceftriaxone
b. Gentamicin and Ceftriaxone
c. Vancomycin,Ampicillin and Ceftriaxone
d. Cefepime and Vancomycin
Ans-287: (c) Vancomycin,Ampicillin and Ceftriaxone
Ref: Read the text below
Sol:
Ampicillin should be added to the empirical regimen for coverage of Listeria
monocytogenes in individuals <3 months of age, those >55, or those with suspected
impaired cell-mediated immunity because of chronic illness, organ transplantation,
pregnancy, malignancy, or immunosuppressive therapy.
Peculiarities of Listeria meningitis:
1)A sub acute presentation when compared to acute meningitis due to other bacterial
agents
2)Nuchal rigidity and meningeal signs are less common.
3)Focal findings and seizures are common
4)CSF neutrophil predominance is modest when compared to other bacterial meningitis
5)Direct invasion of parenchyma can lead to cerebritis and focal abscess
6)Invasion of brain stem can cause rhombencephalitis

Q-288 All the following statements regarding the nontypable strains of Haemophilus
influenzae is correct, except:
a. It is encapsulated
b. It is the most common bacterial cause of infective exacerbation of COPD
c. There is no vaccines available currently
d. Causes mucosal infections like otitis media by contiguous spread
Ans-288: (a) It is encapsulated
Ref: Read the text below
Sol:
Differences between haemophilus influenzae b and non-typable strains of haemophilus
influenzae is summarized as :
Feature
Type b Strains
Nontypable strains
Capsule
Made of Ribosyl-ribitol
Non capsulated
phosphate
Pathogenesis
Invasive infections due to
Mucosal infections due to

www.damsdelhi.com

Email: info@damsdelhi.com

DAMS CBT 2016 SET - 1

Clinical manifestations

Vaccine

hematogenous spread
Meningitis and invasive
infections in incompletely
immunized infants and
children
Highly effective conjugate
vaccines

contiguous spread
Otitis media in infants and
children; lower respiratory
tract infections in adults with
chronic bronchitis
Not available

Nontypable H. influenzae is the most commonbacterial cause of exacerbations of COPD

Q-289 All the following features in a patient with pneumonia may suggest Legionella
pneumophila as the possible etiological agent, except:
a. Diarrhea
b. Hyponatremia
c. Good response to beta-lactam agents
d. Numerous neutrophils but no organisms revealed by Grams staining of respiratory
secretions
Ans-289: (c) Good response to beta-lactam agents
Ref: Read the text below
Sol:
The following features in a patient with pneumonia may be suggestive of Legionella as the
possible etiological agent
Diarrhea
High fever (>40C; >104F)
Numerous neutrophils but no organisms revealed by Grams staining of respiratory
secretions
Hyponatremia (serum sodium level <131 mg/dL)
Failure to respond to -lactam drugs (penicillins or cephalosporins) and aminoglycoside
antibiotics
Occurrence of illness in an environment in which the potable water supply is known to
be contaminated with Legionella
Onset of symptoms within 10 days after discharge from the hospital (hospital-acquired
legionellosis manifesting after discharge or transfer)

Q-290 Which of the following antibiotics has the weakest association with the
development of Clostridium difficile associated disease?
a. Ceftriaxone
b. Ciprofloxacin
c. Moxifloxacin
d. Piperacillin/Tazobactum
Ans-290: (d) Piperacillin/Tazobactum
Ref: Read the text below
Sol:

www.damsdelhi.com

Email: info@damsdelhi.com

DAMS CBT 2016 SET - 1

Clindamycin was the first antibiotics associated with CDI.


The second- and third-generation cephalosporins, particularly cefotaxime, ceftriaxone,
cefuroxime, and ceftazidime, are frequently responsible for this condition, and the
fluoroquinolones (ciprofloxacin, levofloxacin, and moxifloxacin) are the most recent drug
class to be implicated in hospital outbreaks.
Penicillin/ -lactamase-inhibitor combinations such as ticarcillin/clavulanate and
piperacillin/tazobactam pose significantly less risk.
However, all antibiotics, including vancomycin and metronidazole (the agents most
commonly used to treat CDI), carry a risk of subsequent CDI.
Rare cases are reported in patients without prior antibiotic exposure.
Q-291 Inprosapagnosia, lesion is in
a. Temporo-occipital junction
b. Temporo-parietal junction
c. Parieto-occipital junction2
d. None of the above
Ans 291: (a) Temporo-occipital junction
Ref: Read the text below
Sol:
Kaplan synopsis textbook of psychiatry quotes In humans, the clinical syndrome of
prosopagnosia describes the inability to recognize faces, in the presence of preserved
recognition of other environmental objects.
On the basis of pathological and radiological examination of individual patients,
prosopagnosia is thought to result from disconnection of the left ITC from the visual
association area in the left parietal lobe.

Q-292 Mechanism of action of atypical antipsychotics?


a. 5HT2C agonism
b. 5HT2C antagonism
c. 5HT2B antagonism
d. 5HT2A antagonism
Ans 292: (d) 5HT2A antagonism
Ref: Read the text below
Sol:
5 HT2A : (agonism) Target of hallucinogens, (antagonism) atypical antipsychotics
5HT2B: Regulation of stomach contraction, 2B agonism : cardiac valvulopathy
5HT2C : Regulation of appetite,anxiety,seizures
Metabolic side effects
Q-293 All are included under Date Rape Drugs except?
a. GHB
b. Ketamine
www.damsdelhi.com

Email: info@damsdelhi.com

DAMS CBT 2016 SET - 1

c. Flunitrazepam
d. Ecstasy
Ans 293: (d) Ecstasy
Ref: Read the text below
Sol:
The use of a certain group of substances popularly called club drugs is often associated
with dance clubs, bars, and all-night dance parties (raves).
The group includes LSD, -hydroxybutyrate (GHB), ketamine, methamphetamine,
MDMA (ecstasy), and Rohypnol or roofies (flunitrazepam).
These substances are not all in the same drug class, nor do they produce the same
physical or subjective effects. GHB, ketamine, and Rohypnol have been called date rape
drugs because they produce disorienting and sedating effects, and often users cannot recall
what occurred during all or part of an episode under the influence of the drug.
Hence, it is alleged that these drugs might be surreptitiously placed in a beverage, or a
person might be convinced to take the drug and then not recall clearly what occurred after
ingestion.

Q-294 The patient shown in the image presented with chief complaints of itching over left
foot. This is an example of:

a. Organic hallucination
b. Functional hallucination
c. Illusion
d. Pseudo hallucination
Ans 294: (a) Organic hallucination
Ref: Read the text below
Sol:
www.damsdelhi.com

Email: info@damsdelhi.com

DAMS CBT 2016 SET - 1

The phantom limb is the most common organic somatic hallucination of psychiatric
origin. In this case the patient feels that they have a limb from which in fact they are not
receiving any sensations either because it has been amputated or because the sensory
pathways from it have been destroyed.
In rare cases with thalamoparietal lesions the patient describes a third limb. In most
phantom limbs the phenomenon is produced by peripheral and central disorders.
Phantom limb occurs in about 95% of all amputations after the age of 6 years.
Occasionally a phantom limb develops after a lesion of the peripheral nerve or the medulla
or spinal cord.
The phantom limb does not necessarily correspond to the previous image of the limb in
that it may be shorter or consist only of the distal portion so that the phantom hand arises
from the shoulder.
If there is clouding of consciousness, the patient may be deluded that the limb is real.
Equivalent perceptions of phantom organs may also occur after other surgical
procedures such as mastectomy, enuleation of the eye, removal of the larynx or the
construction of a colostomy.
The person is aware of the existence of the organ or limb and describes pain or
paraesthesia in the space occupied by the phantom organ and this persists in a minority of
patients.
When the experience is related to a limb the perception shrinks over time, with distal
parts disappearing more quickly than those that are proximal. Lesions of the parietal lobe
can also produce somatic hallucinations with distortion or splitting-off of body parts.

Q-295 6 yr old boy presented with bed wetting at night but dryness by day. Urine
microscopy normal. No developmental delay.Next step?
a. Refer to child psychiatrist
b. Reassurance and follow up after 6 months
c. Complete blood count
d. USG Kidney and Bladder
Ans 295: (d) USG Kidney and Bladder
Ref: Read the text below
Sol:
"To make the diagnosis of enuresis, organic causes of bladder dysfunction must be
investigated and ruled out.
Organic syndromes, such as urinary tract infections, obstructions, or anatomical
conditions, are found most often in children who experience both nocturnal and diurnal
enuresis combined with urinary frequency and urgency"

Q-296 A patient presented with low mood, lethargy, easy fatigue ability, lack of enjoyment
in life for 2 months, with no past history of similar symptoms, no past history of medical or
surgical illness. He was started on escitalopram 10 mg, 1 tablet in morning after breakfast
by the psychiatrist. On 4th day of treatment, patient returns with complaints of worsening

www.damsdelhi.com

Email: info@damsdelhi.com

DAMS CBT 2016 SET - 1

of mood symptoms, some anxiety symptoms, and saying that no relief from medication,
what should be the next step?
a. Increase the dose of escitalopram
b. Substitute escitalopram with another antidepressant belonging to different mechanism
of action
c. Augment escitalopram with another antidepressant
d. Reassure the patient that the drug will show the effect, and no change needed at present
Ans 296: (d) Reassure the patient that the drug will show the effect, and no change
needed at present
Ref: Read the text below
Sol:
SSRI are the drugs of first line in treatment of depression, and they usually take upto 2-3
weeks to show the therapeutic response (at least 1 week).
Patient sometimes may have episodic exacerbations or worsening of symptoms, where
he is reassured that the drug will show action within 2-3 weeks time.

Q-297 Which metabolite of nicotine is observed in urine of passive smokers?


a. Cotinine
b. Anabasine
c. Nornicotine
d. Polycyclic aromatic hydrocarbons
Ans 297: (a) Cotinine
Ref: Read the text below
Sol:
Urine cotinine has been observed to accumulate up to 20 ng/mL from passive exposure.
Neither anabasine nor nornicotine accumulates from passive exposure.

Q-298 Which of the following best differentiates delirium from dementia?


a. Impairment of memory
b. Global cognitive impairment
c. Fluctuating in nature
d. Psychomotor hyperactivity
Ans 298: (c) Fluctuating in nature
Ref: Read the text below
Sol:
Although the best to differentiate is altered sensorium but thats not in options here.
The hallmark symptom of delirium is an impairment of consciousness, usually occurring
in association with global impairments of cognitive functions.
Frequency of Clinical Features of Delirium Contrasted with Dementia
Feature
Dementia
Delirium
Onset
Slow
Rapid
Duration
Months to years
Hours to weeks
Attention
Preserved
Fluctuates

www.damsdelhi.com

Email: info@damsdelhi.com

DAMS CBT 2016 SET - 1

Memory
Impaired remote memoryImpaired recent and immediate memory
Speech
Word-finding difficulty Incoherent (slow or rapid)
Sleep wake cycleFragmented sleep
Frequent disruption (e.g., day night reversal)
Thoughts
Impoverished
Disorganized
Awareness
Unchanged
Reduced
Alertness
Usually normal
Hypervigilant or reduced vigilance

Q-299 Rett's syndrome is characterized by all except:


a. Regression of acquired skills
b. Breath holding spells
c. Autistic behavior
d. Macrocephaly
Ans 299: (d) Macrocephaly
Ref: Read the text below
Sol:
In Retts syndrome, during the first 5 months after birth, infants have age-appropriate
motor skills, normal head circumference, and normal growth. Social interactions show the
expected reciprocal quality.
At 6 months to 2 years of age, however, these children develop progressive
encephalopathy with a number of characteristic features.
The signs often include the loss of purposeful hand movements, which are replaced by
stereotypic motions, such as hand-wringing; the loss of previously acquired speech;
psychomotor retardation; and ataxia.
Other stereotypical hand movements may occur, such as licking or biting the fingers
and tapping or slapping. The head circumference growth decelerates and produces
microcephaly.
All language skills are lost, and both receptive and expressive communicative and social
skills seem to plateau at developmental levels between 6 months and 1 year.
Poor muscle coordination and an apraxic gait with an unsteady and stiff quality develop.
All of these clinical features are diagnostic criteria for the disorder.
Associated features include seizures in up to 75 percent of affected children and
disorganized EEGs with some epileptiform discharges in almost all young children with
Rett's disorder, even in the absence of clinical seizures.
An additional associated feature is irregular respiration, with episodes of
hyperventilation, apnea, and breath holding. The disorganized breathing occurs in most
patients while they are awake; during sleep, the breathing usually normalizes. Many
patients with Rett's disorder also have scoliosis.
As the disorder progresses, muscle tone seems to change from an initial hypotonic
condition to spasticity to rigidity.

www.damsdelhi.com

Email: info@damsdelhi.com

DAMS CBT 2016 SET - 1

Q-300 Following brain regions involved, as shown in image is related to pathophysiology


of which psychiatric disorder?

a.
b.
c.
d.

OCD
Depression
Dementia
Schizophrenia

Ans 300: (a) OCD


Ref: Read the text below
Sol:
Neuroimaging in patients with OCD has produced converging data implicating altered
function in the neurocircuitry between orbitofrontal cortex, caudate, and thalamus.
Various functional brain-imaging studies for example, positron emission tomography
(PET) have shown increased activity (e.g., metabolism and blood flow) in the frontal lobes,
the basal ganglia (especially the caudate), and the cingulum of patients with OCD.
The involvement of these areas in the pathology of OCD appears more associated with
corticostriatal pathways than with the amygdala pathways that are the current focus of
much anxiety disorder research.
Pharmacological and behavioral treatments reportedly reverse these abnormalities.
Data from functional brain-imaging studies are consistent with data from structural
brain-imaging studies. Both computed tomographic (CT) and magnetic resonance imaging
(MRI) studies have found bilaterally smaller caudates in patients with OCD.

www.damsdelhi.com

Email: info@damsdelhi.com

DAMS CBT 2016 SET - 1

Both functional and structural brain-imaging study results are also compatible with the
observation that neurological procedures involving the cingulum are sometimes effective
in the treatment of OCD.
One recent MRI study reported increased T1 relaxation times in the frontal cortex, a
finding consistent with the location of abnormalities discovered in PET studies.

www.damsdelhi.com

Email: info@damsdelhi.com

Você também pode gostar